45
2016 Pharmacotherapy Specialty Examination Review Course: Complex Breast Cancer Case Helen M. Marshall, Pharm.D., BCPS, BCOP Clinical Pharmacist, Hematology/Oncology Seattle Cancer Care Alliance/University of Washington Medical Center Clinical Assistant Professor University of Washington School of Pharmacy Seattle, Washington Learning Objectives: At the end of the presentation, the pharmacist should be able to: Select the appropriate treatment and monitoring of a complex patient with multiple conditions, including breast cancer, febrile neutropenia, and pulmonary embolism. Compare and contrast current breast cancer screening recommendations. Develop a plan to manage pain and nausea/vomiting in a patient with cancer. Determine how to manage drugdrug and drugdisease interactions in a cancer patient. Discuss safety issues in this population. Identify and recommend appropriate resource organizations/groups to assist a specific patient. Format: This session will use a series of audience response questions to engage the audience and to prepare participants to answer similar questions on the pharmacotherapy board certification examination. The speaker will discuss drug therapy management in a cancer patient and discuss the rationale and guidelines driving therapeutic decisions. Premise: You are a clinical pharmacist in a 450bed institution. You also participate in a continuity clinic one afternoon per week in an ambulatory, multidisciplinary breast cancer clinic. You are responsible for drug therapy monitoring in both settings and ensuring patient safety for both chemotherapy and supportive care. You have online access to an integrated medical record that is shared in both the inpatient and outpatient settings. NOTE: NCCN Clinical Practice Guidelines in Oncology (NCCN Guidelines®) are referenced with permission from the National Comprehensive Cancer Network® (NCCN®). To view the most recent and complete version of the guideline, go online to NCCN.org. NATIONAL COMPREHENSIVE CANCER NETWORK®, NCCN®, NCCN GUIDELINES®, and all other NCCN Content are trademarks owned by the National Comprehensive Cancer Network, Inc. ©2016 American Society of Health-System Pharmacists, Inc. All rights reserved. ____________________________________________________________________________________________________________ 1

2016 Pharmacotherapy Specialty Examination Review Course ...elearning.ashp.org/Files/Org/c52850f8e2e14ca8b14b... · 2016 Pharmacotherapy Specialty Examination Review Course: Complex

  • Upload
    others

  • View
    8

  • Download
    0

Embed Size (px)

Citation preview

Page 1: 2016 Pharmacotherapy Specialty Examination Review Course ...elearning.ashp.org/Files/Org/c52850f8e2e14ca8b14b... · 2016 Pharmacotherapy Specialty Examination Review Course: Complex

2016 Pharmacotherapy Specialty Examination Review Course:  

Complex Breast Cancer Case 

Helen M. Marshall, Pharm.D., BCPS, BCOP 

Clinical Pharmacist, Hematology/Oncology 

Seattle Cancer Care Alliance/University of Washington Medical Center 

Clinical Assistant Professor 

University of Washington School of Pharmacy 

Seattle, Washington 

Learning Objectives: 

At the end of the presentation, the pharmacist should be able to: 

Select the appropriate treatment and monitoring of a complex patient with multiple conditions,

including breast cancer, febrile neutropenia, and pulmonary embolism.

Compare and contrast current breast cancer screening recommendations.

Develop a plan to manage pain and nausea/vomiting in a patient with cancer.

Determine how to manage drug‐drug and drug‐disease interactions in a cancer patient.

Discuss safety issues in this population.

Identify and recommend appropriate resource organizations/groups to assist a specific patient.

Format:  This session will use a series of audience response questions to engage the audience and to 

prepare participants to answer similar questions on the pharmacotherapy board certification 

examination. The speaker will discuss drug therapy management in a cancer patient and discuss the 

rationale and guidelines driving therapeutic decisions.   

Premise:  You are a clinical pharmacist in a 450‐bed institution. You also participate in a continuity clinic 

one afternoon per week in an ambulatory, multidisciplinary breast cancer clinic. You are responsible for 

drug therapy monitoring in both settings and ensuring patient safety for both chemotherapy and 

supportive care. You have online access to an integrated medical record that is shared in both the 

inpatient and outpatient settings.   

NOTE:  NCCN Clinical Practice Guidelines in Oncology (NCCN Guidelines®) are referenced with 

permission from the National Comprehensive Cancer Network® (NCCN®). To view the most recent and 

complete version of the guideline, go online to NCCN.org. NATIONAL COMPREHENSIVE CANCER 

NETWORK®, NCCN®, NCCN GUIDELINES®, and all other NCCN Content are trademarks owned by the 

National Comprehensive Cancer Network, Inc. 

©2016 American Society of Health-System Pharmacists, Inc. All rights reserved.

____________________________________________________________________________________________________________ 1

Page 2: 2016 Pharmacotherapy Specialty Examination Review Course ...elearning.ashp.org/Files/Org/c52850f8e2e14ca8b14b... · 2016 Pharmacotherapy Specialty Examination Review Course: Complex

  

PATIENT CASE      Date: March 1, 2016 

Initials LK  DOB/Age 

54 

Sex 

Race/Ethnicity 

Chinese  

Source  

Patient and medical 

records 

Chief Complaint/History of Present Illness: 

“My husband felt a lump in my right breast.” 

 

The patient presents to a multidisciplinary breast cancer clinic at your comprehensive cancer center 

to establish a treatment plan for her newly diagnosed right breast cancer.   

 

 

Past Medical History 

None 

From Medical Record 

 

Breast Cancer, Stage IIB 

• Diagnosed March 2016 in her right breast 

• Stage IIB (T2 N1 M0) 

• ER and PR positive 

• HER2 negative 

• Postmenopausal (menopause at age 51) 

 

Breast Cancer Treatment 

• Surgery 

▫ Mastectomy with axillary lymph node dissection 

▫ 4 positive lymph nodes 

• Radiation 

▫ Chest wall and regional lymph nodes 

• Chemotherapy 

▫ Dose‐dense AC (Adriamycin [doxorubicin] and cyclophosphamide) 

▫ Doxorubicin 60 mg/m2 IV day 1 

▫ Cyclophosphamide 600 mg/m2 IV day 1 

▫ Every 14 days X 4 cycles 

▫ Dose‐dense AC requires growth factor support 

▫ Followed by paclitaxel  

▫ Paclitaxel 80 mg/m2 IV day 1 weekly X 12 cycles 

 

 

 

   

©2016 American Society of Health-System Pharmacists, Inc. All rights reserved.

____________________________________________________________________________________________________________ 2

Page 3: 2016 Pharmacotherapy Specialty Examination Review Course ...elearning.ashp.org/Files/Org/c52850f8e2e14ca8b14b... · 2016 Pharmacotherapy Specialty Examination Review Course: Complex

  

Current Prescription/OTC Medications       

Start Date  Drug Name/Strength/Regimen  Indication 

6/2016  Ondansetron 8 mg orally every 8 hr prn  Nausea/vomiting 

6/2016  Prochlorperazine 10 mg orally every 6 hr 

prn 

Nausea/vomiting 

6/2016  Lorazepam 0.5‐1 mg orally every 6 hr prn  Nausea/vomiting; anxiety 

6/2016  Palonosetron 0.25 mg IV and 

dexamethasone 10 mg IV every 2 weeks 

pre‐chemotherapy 

Prevention of nausea and 

vomiting  

6/2016  Doxorubicin 60 mg/m2 IV and 

cyclophosphamide 60 mg/m2 IV every 2 

weeks 

Adjuvant breast cancer 

treatment 

Vaccinations:  Influenza vaccine fall 2015 Pharmacy(ies) Used: local 

community pharmacy, 

cancer center 

RX Payment: Private (15% co‐insurance)  Meds Admin by: Self  

Drug Allergies/Adverse Effects:  NKDA 

 

Family Medical History:  Father: MI s/p CABG X 2 at age 55; Mother has a history of stage I 

breast cancer s/p mastectomy with reconstruction and no recurrence; Brother has MS 

 

Social 

History 

Residence: lives at home w/ 

husband 

Occupation:  None

Smoking:   1 pack per day from age 18 to 30  EtOH:  None 

Illicit Drugs:  None  Language: Mandarin speaking, 

minimal English 

Education:  high school in China  Family/Social Environment:  Lives with 

husband; 2 daughters, one in college, 

and one in graduate school for 

electrical engineering 

Review of Systems:  Per HPI /PE 

Height 167 cm, Weight 78 kg, BSA 1.9 m2 

 

   

©2016 American Society of Health-System Pharmacists, Inc. All rights reserved.

____________________________________________________________________________________________________________ 3

Page 4: 2016 Pharmacotherapy Specialty Examination Review Course ...elearning.ashp.org/Files/Org/c52850f8e2e14ca8b14b... · 2016 Pharmacotherapy Specialty Examination Review Course: Complex

  

Presentation Questions  

1. Should LK have previously undergone screening with mammography prior to her diagnosis?  

a. Yes, annually starting at age 20 

b. Yes, due to her family history 

c. No, because LK is under age 55 

d. No, due to her family history 

 

2. Which of the following should be monitored for our patient on this chemotherapy regimen? 

a. CBC, liver function tests, renal function, and neurologic function 

b. CBC, liver function tests, renal function, and signs and symptoms of hemorrhagic cystitis 

c. CBC, liver function tests, and cardiac function 

d. CBC, liver function tests, and signs and symptoms tumor lysis syndrome 

 

3. What is the appropriate antiemetic regimen for LK while receiving treatment with dose‐dense AC? 

a. Ondansetron 8 mg IV and dexamethasone 12 mg IV day 1 followed by dexamethasone 4 mg 

PO BID days 2‐4 

b. Fosaprepitant 150 mg IV, ondansetron 8 mg IV, and dexamethasone 12 mg IV day 1 followed 

by dexamethasone 4 mg PO BID days 2‐4 

c. Fosaprepitant 150 mg IV, ondansetron 8 mg IV, and dexamethasone 12 mg IV day 1 

d. Palonosetron 0.25 mg IV and dexamethasone 10 mg IV day 1 followed by dexamethasone 4 

mg PO BID days 2‐4 

 

4. Which of the following should be initiated as empiric therapy for febrile neutropenia in LK? 

a. Ciprofloxacin 500 mg IV every 8 hours 

b. Ciprofloxacin 500 mg PO every 8 hours and clindamycin 450 mg PO every 8 hours  

c. Ciprofloxacin 500 mg PO every 8 hours and amoxicillin/clavulanate 500 mg PO every 8 

hours 

d. Imipenem 500 mg IV every 6 hours 

 

5. What factors for LK should be considered when deciding if vancomycin should be added to LK’s 

regimen? 

a. All FN patients should receive vancomycin 

b. Presence of a central venous catheter 

c. Prior ciprofloxacin prophylaxis 

d. Reported symptom of cough 

 

6. Which of the following is the best choice for empiric antifungal coverage for LK? 

a. Micafungin 50 mg IV once daily 

b. Micafungin 100 mg IV once daily 

c. Fluconazole 200 mg IV once daily 

d. Voriconazole 6 mg/kg IV every 12 hours 

 

   

©2016 American Society of Health-System Pharmacists, Inc. All rights reserved.

____________________________________________________________________________________________________________ 4

Page 5: 2016 Pharmacotherapy Specialty Examination Review Course ...elearning.ashp.org/Files/Org/c52850f8e2e14ca8b14b... · 2016 Pharmacotherapy Specialty Examination Review Course: Complex

  

7. Which of the following is a risk factor for venous thromboembolism (VTE) in LK? 

a. Metastatic Breast Cancer  

b. Nausea and vomiting 

c. Presence of a Central Venous Catheter 

d. Depression 

 

8. Which of the following therapies should be initiated in LK for her pulmonary embolism? 

a. Warfarin 5 mg orally daily 

b. Apixaban 10 mg orally twice daily 

c. Dalteparin 12,500 units subcut every 24 hr 

d. Enoxaparin 80 mg subcut every 12 hr 

 

9. LK’s oncologist asks you for a conversion to a long‐acting opioid as LK has escalated her 

hydrocodone use and is still reporting poor pain control with inability to sleep.  Which of the 

following is the best regimen for LK?  

a. Fentanyl TDS 100 mcg/hr every 72 hr 

b. Morphine SR 15 mg po every 8 hr 

c. Morphine SR 15 mg po every 12 hr 

d. Morphine SR 30 mg po every 12 hr 

 

10. LK’s oncologist follows your advice and initiates the long acting pain regimen that was 

recommended. Which of the following is the best breakthrough pain regimen for her? 

a. Morphine IR 15 mg po every 1‐2 hr 

b. Morphine IR 15 mg po every 4‐6 hr 

c. Morphine IR 30 mg po every 6 hr 

d. Oxycodone 2.5 mg po every 6 hr  

 

 

11. Which of these therapies is a possible treatment option for LK? 

a. Palbociclib and fulvestrant 

b. Ado‐trastuzumab emtansine  

c. Pertuzumab, trastuzumab, and paclitaxel  

d. Palbociclib and docetaxel 

 

   

©2016 American Society of Health-System Pharmacists, Inc. All rights reserved.

____________________________________________________________________________________________________________ 5

Page 6: 2016 Pharmacotherapy Specialty Examination Review Course ...elearning.ashp.org/Files/Org/c52850f8e2e14ca8b14b... · 2016 Pharmacotherapy Specialty Examination Review Course: Complex

  

Additional Information on Breast Cancer 

 

Febrile Neutropenia Pearls 

 

A clinical practice guideline for the use of antimicrobial agents in patients with febrile neutropenia 

and cancer was published by the Infectious Diseases Society of America (IDSA) in 2011. Some of the 

changes in this guideline include an emphasis on risk stratification of patients with FN,  improved 

guidance on which patients benefit from prophylaxis (antibacterial, antiviral, and antifungal), and 

discussion of the use of empiric vs. preemptive antifungal therapy and emerging resistance 

problems. 

Main algorithms are below: 

 

Frefield AG, Bow EJ, Sepkowitz KA et al. Clinical practice guideline for the use of antimicrobial agents in neutropenic patients 

with cancer: 2010 update by the Infectious Disease Society of America. Clin Infect Dis. 2011; 52:e56‐93. 

http://www.uphs.upenn.edu/bugdrug/antibiotic_manual/idsaneutropenicfever2010.pdf (accessed 2015 May 19). 

 

Clinical infectious diseases by Infectious Diseases Society of America. Reproduced with permission of Oxford University Press in 

the format Journal via Copyright Clearance Center. 

©2016 American Society of Health-System Pharmacists, Inc. All rights reserved.

____________________________________________________________________________________________________________ 6

Page 7: 2016 Pharmacotherapy Specialty Examination Review Course ...elearning.ashp.org/Files/Org/c52850f8e2e14ca8b14b... · 2016 Pharmacotherapy Specialty Examination Review Course: Complex

  

  Frefield AG, Bow EJ, Sepkowitz KA et al. Clinical practice guideline for the use of antimicrobial agents in neutropenic patients 

with cancer: 2010 update by the Infectious Disease Society of America. Clin Infect Dis. 2011; 52:e56‐93. 

http://www.uphs.upenn.edu/bugdrug/antibiotic_manual/idsaneutropenicfever2010.pdf (accessed 2015 May 19). 

 

Clinical infectious diseases by Infectious Diseases Society of America. Reproduced with permission of Oxford University Press in 

the format Journal via Copyright Clearance Center. 

 

 

Modifications to initial antibiotics based on resistance. 

o MRSA  consider early addition of vancomycin, linezolid or daptomycin 

o VRE  consider early addition of linezolid or daptomycin 

o ESBLs  consider early use of a carbapenem 

o KPCs  consider early use of polymyxin‐colistin or tigecycline 

Duration of therapy is dictated by organism/site if documented infection; or at least until ANC 

recovers (>0.5 thou/microL) or longer depending on clinical situation. 

©2016 American Society of Health-System Pharmacists, Inc. All rights reserved.

____________________________________________________________________________________________________________ 7

Page 8: 2016 Pharmacotherapy Specialty Examination Review Course ...elearning.ashp.org/Files/Org/c52850f8e2e14ca8b14b... · 2016 Pharmacotherapy Specialty Examination Review Course: Complex

  

ASCO published guidelines for antimicrobial prophylaxis and outpatient management of FN 

o In patients with FN, those that meet MASCC criteria, those in Talcott group 4, and without 

additional risk factors may be managed with empiric fluoroquinolone and 

amoxicillin/clavulanate (clindamycin for patients with a penicillin allergy)  

NCCN® also lists moxifloxacin as an outpatient option for low risk FN patients 

Fluoroquinolone prophylaxis may be used for patients at high risk with expected duration of 

neutropenia  ≤ 0.1 thou/microL for > 7 days. 

Empiric antifungal coverage for persistent/recurrent fever after 4‐7 days of antibiotics for 

neutropenia expected to be > 7 days; preemptive therapy is acceptable for high‐risk patients. 

Antifungal prophylaxis should be used  

o Against Candida for allogeneic stem cell transplant patients or undergoing intensive 

reinduction or salvage chemotherapy for acute leukemia (fluconazole, itraconazole, 

voriconazole, micafungin or caspofungin) 

o Against Aspergillus for patients ≥ 13 years of age undergoing intensive therapy for acute 

leukemia or myelodysplastic syndrome with high risk of invasive aspergillosis (use 

posaconazole).  May also use antifungal prophylaxis pre‐engraftment in transplant patients 

with history of invasive aspergillosis, prolonged neutropenia ≥ 14 days, or prolonged period 

of neutropenia prior to transplant. 

Antiviral Prophylaxis 

o Acyclovir for herpes simplex virus (HSV)‐seropositive patients undergoing allogeneic stem 

cell transplant or leukemia induction therapy 

o Yearly influenza vaccination for all patients being treated for cancer 

o No other routine prophylaxis, but appropriate treatment if infection develops 

Myeloid growth factors are not recommended for established fever and neutropenia 

Central line‐associated bloodstream infections (CLABSI) in patients with FN 

o Differential time to positivity  > 120 minutes after blood cultures drawn from both the 

central line and peripherally suggests a CLABSI 

o Infections caused by Staphylococcus aureus, Pseudomonas aeruginosa, fungi or 

mycobacteria 

Remove the catheter 

Systemic therapy for at least 14 days 

o If caused by coagulase‐negative Staphylococci, may maintain central line and treat with 

systemic therapy with or without antibiotic lock therapy 

o Use duration of therapy of 4‐6 weeks for deep tissue infection, endocarditis, septic 

thrombosis or persistent bacteremia/fungemia > 72 hours after catheter removal 

IDSA guidelines are also available (see reference list) for the treatment of candidiasis (2009) and 

catheter‐related infections (2009). 

 

   

©2016 American Society of Health-System Pharmacists, Inc. All rights reserved.

____________________________________________________________________________________________________________ 8

Page 9: 2016 Pharmacotherapy Specialty Examination Review Course ...elearning.ashp.org/Files/Org/c52850f8e2e14ca8b14b... · 2016 Pharmacotherapy Specialty Examination Review Course: Complex

  

Anticoagulation Pearls 

 

Oncology patients should be provided education about the signs and symptoms of VTE 

Anticoagulation should not be used to extend survival in patients with cancer in the absence of 

other indications 

At this time, there are insufficient data to recommend the use of dabigatran, rivaroxaban, apixaban 

and edoxaban in oncology patients.  In 2016, the American College of Chest Physicians provided a 

guideline update on antithrombotic therapy for venous thromboembolism disease that suggests low 

molecular weight heparin (LMWH) over these newer agents for long‐term treatment (first 3 months) 

of cancer‐associated thrombosis, with an evidence level of 2C.   

Low molecular weight heparin (LMWH) remains the cornerstone of therapy. 

Anti‐factor Xa monitoring is not routinely recommended, but it is used by some clinicians. The goal 

trough concentration is < 0.5 units/mL. You may consider this type of monitoring in special 

situations, such as changing renal function, pregnancy, or obesity. 

Routine monitoring of LMWH should include weight, SCr/CrCl, hematocrit and platelets.  

In patients receiving extended anticoagulant therapy, the need for continued treatment should be 

periodically reevaluated. 

In patients with active cancer, extended anticoagulant therapy is recommended (beyond 3 months 

of therapy) even when the risk for bleeding is high. 

In patients with acute pulmonary embolism (PE) treated with LMWH, use a once‐daily regimen 

instead of a twice‐daily regimen if possible to minimize the number of injections. 

In patients with upper extremity DVT (UEDVT) associated with a central venous catheter, the 

catheter should not be removed if it is functional and there is an ongoing need for it.   

In cancer patients with UEDVT in whom the catheter is removed, 3 months of anticoagulant therapy 

is suggested over extended therapy (Grade 2C). 

In cancer patients with UEDVT in whom the catheter is not removed, anticoagulation should 

continue for as long as the catheter remains (at least 3 months). 

Hospitalized patients with cancer should receive prophylactic anticoagulation unless there is a 

contraindication. Prophylactic anticoagulant therapy is recommended in the postoperative setting 

for 4 weeks, especially for high‐risk abdominal or pelvic cancer surgery. 

Prophylactic anticoagulant therapy is recommended in high‐risk ambulatory settings 

o Multiple myeloma patients receiving lenalidomide or thalidomide in combination with high‐

dose dexamethasone or a multi‐agent doxorubicin‐containing regimen or multiple myeloma 

patients with other risk factors 

Prophylactic LMWH (i.e., enoxaparin 40 mg subcutaneously every 24 hours) 

Warfarin to target international normalized ratio (INR) 2‐3 

o Low‐risk myeloma patients may receive prophylaxis with aspirin 81‐325 mg once daily 

o Select the prophylactic agent based on cost, Food and Drug Administration (FDA)‐approved 

indication, ease of administration, reversibility, monitoring requirements, and presence of 

renal failure 

In the setting of heparin‐induced thrombocytopenia (HIT), treatment with a direct thrombin 

inhibitor (DTI) or fondaparinux should overlap with warfarin therapy for a minimum of 5 days, and 

be continued until the INR is >2 for at least 24 hours. 

©2016 American Society of Health-System Pharmacists, Inc. All rights reserved.

____________________________________________________________________________________________________________ 9

Page 10: 2016 Pharmacotherapy Specialty Examination Review Course ...elearning.ashp.org/Files/Org/c52850f8e2e14ca8b14b... · 2016 Pharmacotherapy Specialty Examination Review Course: Complex

  

If argatroban is used, the target INR should be 4 before the drug is discontinued, and then the INR 

should be repeated within 3‐6 hours after discontinuation of argatroban. 

Chromogenic factor X could be used to monitor the INR during DTI therapy because it is not affected 

by the DTI. 

Patients with cancer should be periodically assessed for venous thromboembolism (VTE) risk 

Risk factors for VTE in malignancy 

o General Patient 

Active cancer 

Advanced stage of cancer 

Types: brain, pancreas, stomach, bladder, gynecologic, lung, lymphoma, 

myeloproliferative disease, kidney, metastatic 

Regional bulky lymphadenopathy with extrinsic vascular compression 

Familial and/or acquired hypercoagulability 

Medical comorbidities, including infection, renal disease, pulmonary disease, 

congestive heart failure, or arterial thromboembolism 

Poor performance status 

Advanced age 

o Treatment‐related risk factors 

Major surgery 

Central catheter/IV catheter 

Chemotherapy, especially bevacizumab or thalidomide/lenalidomide with 

dexamethasone 

Exogenous estrogen, including hormone‐replacement therapy, contraceptives, 

tamoxifen/raloxifene, diethylstilbestrol 

o Modifiable risk factors 

Smoking 

Obesity 

Exercise (lack of) 

o Multiple myeloma patient risk factors 

M spike > 1.6 g/dL 

Hyperviscosity 

Progressive disease 

o High‐risk outpatients on chemotherapy (based on risk factor combinations) 

Active cancer with high risk of DVT (See above) 

Pre‐chemotherapy platelet count > 300 thou/microL 

Pre‐chemotherapy WBC > 11 thou/microL 

Hemoglobin <10 g/dL 

Use of erythropoiesis‐stimulating agents 

BMI ≥ 35 kg/m2 

Prior VTE 

NCCN® also has recommendations for managing therapeutic failure of anticoagulation, which does 

occur in the oncology population. 

 

©2016 American Society of Health-System Pharmacists, Inc. All rights reserved.

____________________________________________________________________________________________________________ 10

Page 11: 2016 Pharmacotherapy Specialty Examination Review Course ...elearning.ashp.org/Files/Org/c52850f8e2e14ca8b14b... · 2016 Pharmacotherapy Specialty Examination Review Course: Complex

  

Chemotherapy Induced Nausea and Vomiting (CINV) Pearls 

 

Risk factors for CINV include 

o Young age 

o Sex (female increases risk) 

o Expectation of severe nausea 

o Chemotherapy 

Dose 

Emetogenicity 

o History of alcohol intake 

There are three main sets of guidelines utilized in clinical practice (MASCC, ASCO, and NCCN®, 

see references).  There are some differences in each guideline in regards to classification of 

emetogenicity risk of each chemotherapeutic agent as well as recommendations for 

prophylaxis/treatment. 

Definitions: Emetic Risk Groups 

o High = Risk in > 90% of patients; cisplatin is a key example and frequently used in clinical 

trials 

o Moderate = Risk in 30% to 90% of patients 

o Low = Risk in 10% to 30% of patients 

o Minimal – Risk in <10% of patients 

The combination of anthracycline and cyclophosphamide was reclassified to highly emetogenic 

in the 2011 update of the ASCO guideline. 

The guidelines also provide guidance for radiation‐induced nausea/vomiting. 

Acute emesis occurs within 24 hours of treatment 

Delayed emesis occurs at least 24 hours after treatment 

Antiemetic treatment for patients receiving combination chemotherapy should be determined 

based on the agent with the highest emetic risk. 

NCCN® Principles of Emesis Control 

o Prevention is the goal 

o Risk of N/V for patients receiving highly or moderately emetogenic regimens lasts 3 days 

for high regimens and 2 days for moderate regimens; use prophylaxis throughout 

o Oral and intravenous 5HT3 antagonists are equally effective at appropriate doses 

o Consider the toxicity of the selected agents (e.g., constipation with 5HT3 antagonists) 

o Choice of antiemetic should be based on regimen emetogenicity, patient factors and 

prior antiemetic history 

o Consider other factors causing N/V in patients with malignancy 

o Consider H2 antagonist or PPI use to decrease dyspepsia which can exacerbate nausea 

o Lifestyle measures (e.g., small, frequent meals) may be useful 

 

   

©2016 American Society of Health-System Pharmacists, Inc. All rights reserved.

____________________________________________________________________________________________________________ 11

Page 12: 2016 Pharmacotherapy Specialty Examination Review Course ...elearning.ashp.org/Files/Org/c52850f8e2e14ca8b14b... · 2016 Pharmacotherapy Specialty Examination Review Course: Complex

  

Recommendations for Prophylaxis 

Emetic Risk Category  Acute  Delayed Notes 

High  NK1 Antagonist AND

5HT3 Antagonist AND 

Dexamethasone 

Dexamethasone

(and aprepitant if used) 

NCCN®: 

Consider adding 

lorazepam, H2 

antagonist or PPI; also 

provides an olanzapine‐

based regimen 

Moderate  Palonosetron AND

Dexamethasone 

Dexamethasone NCCN® notes as above

Low  5HT3 Antagonist OR

Dexamethasone OR 

Dopamine Receptor 

Antagonist 

No routine prophylaxis NCCN®: Consider adding 

lorazepam, H2 

antagonist or PPI 

ASCO: dexamethasone is 

recommended 

Minimal  No routine prophylaxis No routine prophylaxis  

 

Dexamethasone and 5HT3 antagonist are both recommended for high‐dose chemotherapy 

(frequently used in bone marrow transplant) 

The ASCO guideline focuses on intravenous therapy; oral chemotherapy is discussed in 

NCCN® and MASCC 

Two newer oral agents approved for highly and moderately emetogenic chemotherapy 

regimens include rolapitant 180 mg (NK1 antagonist) and netupitant 300 mg/palonosetron 

0.5mg  (combination NK1 antagonist and 5HT3 antagonist). 

 

Highly emetogenic intravenous agents are listed in the slide set 

 

   

©2016 American Society of Health-System Pharmacists, Inc. All rights reserved.

____________________________________________________________________________________________________________ 12

Page 13: 2016 Pharmacotherapy Specialty Examination Review Course ...elearning.ashp.org/Files/Org/c52850f8e2e14ca8b14b... · 2016 Pharmacotherapy Specialty Examination Review Course: Complex

  

Moderately Emetogenic Agents 

  MASCC  NCCN® ASCO 

Intravenous 

Agents 

Oxaliplatin

Cytarabine > 1000 mg/m2 

Carboplatin 

Ifosfamide 

Cyclophosphamide < 

1500 mg/m2 

Azacitidine 

Alemtuzumab 

Doxorubicin 

Daunorubicin 

Epirubicin 

Idarubicin 

Irinotecan 

Bendamustine 

Clofarabine 

 

 

Aldesleukin > 12‐15  

million IU/m2 

Amifostine >300 mg/m2 

Arsenic trioxide 

Azacitidine 

Bendamustine 

Busulfan 

Carboplatin 

Carmustine ≤ 250mg/m2 

Clofarabine 

Cyclophosphamide ≤ 

1500 mg/m2 

Cytarabine > 200 mg/m2 

Dactinomycin 

Daunorubicin 

Doxorubicin < 60 mg/m2 

Epirubicin ≤ 90mg/m2 

Idarubicin 

Ifosfamide < 2000 

mg/m2/dose 

Interferon alfa > 10 

million IU/m2 

Irinotecan 

Melphalan 

Methotrexate ≥ 250 

mg/m2 

Oxaliplatin 

Temozolomide 

Azacitidine 

Alemtuzumab 

Bendamustine 

Carboplatin 

Clofarabine 

Cyclophosphamide < 

1500 mg/m2 

Cytarabine > 1000 mg/m2 

Daunorubicin 

Doxorubicin 

Epirubicin 

Idarubicin 

Ifosfamide 

Irinotecan 

Oxaliplatin 

Oral Agents  Cyclophosphamide

Temozolomide 

Vinorelbine 

Imatinib 

Moderate to High:

Altretamine 

Busulfan (≥ 4 mg/day) 

Crizotinib 

Cyclophosphamide (≥100 

mg/m2/day) 

Estramustine 

Etoposide 

Lomustine (single day) 

Mitotane 

Procarbazine 

Temozolomide (≥75 

mg/m2/day) 

Vismodegib 

Ceritinib 

Lenvatinib 

Olaparib 

Panobinostat 

 

©2016 American Society of Health-System Pharmacists, Inc. All rights reserved.

____________________________________________________________________________________________________________ 13

Page 14: 2016 Pharmacotherapy Specialty Examination Review Course ...elearning.ashp.org/Files/Org/c52850f8e2e14ca8b14b... · 2016 Pharmacotherapy Specialty Examination Review Course: Complex

  

 

 

Low Emetogenicity Agents 

  MASCC  NCCN® ASCO 

Intravenous Agents  Paclitaxel 

Docetaxel 

Mitoxantrone 

Topotecan 

Etoposide 

Pemetrexed 

Methotrexate 

Doxorubicin HCl 

liposome injection 

Temsirolimus 

Ixabepilone 

Mitomycin 

Gemcitabine 

Cytarabine < 1000 mg/m2

5‐fluorouracil 

Bortezomib 

Cetuximab 

Trastuzumab 

Catumaxomab 

Panitumumab 

Temsirolimus 

 

 

Ado‐trastuzumab 

emtansine 

Amifostine ≤ 300mg/m2 

Aldesleukin ≤ 12 million 

IU/m2 

Brentuximab 

Cabazitaxel 

Carfilzomib 

Cytarabine (low dose) 

100‐200 mg/m2 

Docetaxel 

Doxorubicin (liposomal) 

Eribulin 

Etoposide 

5‐fluorouracil 

Floxuridine 

Gemcitabine 

Interferon alfa >5 <10 

million international 

units/m2 

Methotrexate >50 mg/m2 

<250 mg/m2 

Mitomycin 

Mitoxantrone 

Omacetaxine 

Paclitaxel 

Paclitaxel‐albumin 

Pemtrexed 

Pentostatin 

Pralatrexate 

Romidepsin 

Thiotepa 

Topotecan 

Ziv‐aflibercept 

Belinostat 

Blinatumomab 

5‐fluorouracil 

Bortezomib 

Cabazitaxel 

Catumaxomab 

Cytarabine ≤ 1000 mg/m2

Docetaxel 

Doxorubicin (liposomal) 

Etoposide 

Gemcitabine 

Ixabepilone 

Methotrexate 

Mitomycin 

Mitoxantrone 

Paclitaxel 

Panitumumab 

Pemetrexed 

Temsirolimus 

Topotecan 

Trastuzumab 

 

Oral Agents  Capecitabine

Tegafur‐uracil 

Etoposide 

Sunitinib 

Fludarabine 

Everolimus 

Lapatinib 

Low to Minimal:

Axitinib 

Bexarotene 

Bosunitinib 

Busulfan (<4 mg/day) 

Cabozantinib 

Capecitabine 

 

©2016 American Society of Health-System Pharmacists, Inc. All rights reserved.

____________________________________________________________________________________________________________ 14

Page 15: 2016 Pharmacotherapy Specialty Examination Review Course ...elearning.ashp.org/Files/Org/c52850f8e2e14ca8b14b... · 2016 Pharmacotherapy Specialty Examination Review Course: Complex

  

Lenalidomide

Thalidomide 

Chlorambucil

Cyclophosphamide (<100 

mg/m2/day) 

Dasatinib 

Dabrafenib 

Erlotinib 

Everolimus 

Fludarabine 

Gefitinib 

Hydroxyurea 

Imatinib 

Lapatinib 

Lenalidomide 

Melphalan 

Mercaptopurine 

Methotrexate 

Nilotinib 

Pazopanib 

Pomalidomide 

Potaninib 

Regorafenib 

Ruxolitinib 

Sorafenib 

Sunitinib 

Temozolomide (≤75 

mg/m2/day) 

Thalidomide 

Thioguanine 

Topotecan 

Trametanib 

Tretinoin 

Vandetanib 

Vemurafenib 

Vorinostat 

Afatinib 

Ibrutinib 

Idelalisib 

Palbociclib 

 

 

Breakthrough nausea and vomiting 

o Occurs despite prophylaxis (re‐evaluate) 

o Treatment is largely empiric, without strong data or consensus recommendations 

o Drug classes frequently used include phenothiazines and benzodiazepines 

o Consider adding an additional class of medication to the current regimen 

o ASCO suggests olanzapine, high‐dose  intravenous metoclopramide, benzodiazepine 

or dopamine antagonist 

©2016 American Society of Health-System Pharmacists, Inc. All rights reserved.

____________________________________________________________________________________________________________ 15

Page 16: 2016 Pharmacotherapy Specialty Examination Review Course ...elearning.ashp.org/Files/Org/c52850f8e2e14ca8b14b... · 2016 Pharmacotherapy Specialty Examination Review Course: Complex

  

Anticipatory nausea and vomiting 

o Use most active, appropriate regimen for prophylaxis 

o Consider behavioral therapy/desensitization 

o Consider benzodiazepine 

 

Pain Management Pearls 

 

Note that charts and tables of analgesic agents with equianalgesic dosing information are variable. 

Doses recommended were obtained from studies of single doses for acute pain, were not bi‐

directional, and did not account for inter‐patient variability in response.   

In addition to pharmacologic treatment of cancer pain, interventional (e.g., celiac plexus block, 

placement of an intrathecal catheter), behavioral, cognitive, rehabilitative and integrative modalities 

for management of cancer pain are available. 

Prescription drug abuse has continued to rise in the United States, and many states are enacting 

legislation to promote the safe use of opioid analgesics. In many states, cancer patients are exempt 

from legal requirements, but legislation in some states may apply to cancer patients with non‐

cancer chronic pain (e.g., low back pain). 

There is no upper dose limit for opioids in cancer pain. The correct dose controls the patient’s pain 

and does not cause unacceptable side effects. 

Opioids to avoid in patients with cancer pain: meperidine, pentazocine, butorphanol, nalbuphine, 

and buprenorphine. 

Hydrocodone has been reclassified to CII (2014) 

Requirements for risk evaluation and mitigation strategies (REMS) have been established by FDA for 

some opioids, including the newer extended‐release hydromorphone product (available at: 

http://www.exalgorems.com/) and all immediate‐release transmucosal fentanyl products (available 

at: https://www.tirfremsaccess.com/TirfUI/rems/home.action). Prescribers, patients, and 

pharmacies must be registered for outpatient use of these products.   

Conversion to fentanyl transdermal systems (TDS) can be accomplished using morphine equivalents 

(do not use the following table to convert from fentanyl to another opioid):  

 

24‐hr Oral Morphine Dose (mg)  Fentanyl TDS dose (mcg/hr) 

60‐134  25 

135‐224  50 

225‐314  75 

315‐404  100 

405‐494  125 

495‐584  150 

585‐674  175 

675‐764  200 

765‐854  225 

855‐944  250 

945‐1034  275 

1035‐1124  300 

©2016 American Society of Health-System Pharmacists, Inc. All rights reserved.

____________________________________________________________________________________________________________ 16

Page 17: 2016 Pharmacotherapy Specialty Examination Review Course ...elearning.ashp.org/Files/Org/c52850f8e2e14ca8b14b... · 2016 Pharmacotherapy Specialty Examination Review Course: Complex

  

 

Fentanyl TDS Pearls 

o Patients need to be on an oral morphine equivalent of at least 60 mg daily to start using 

transdermal systems 

o The onset of action is approximately 12 hours 

o Removal of the TDS results in continued fentanyl exposure for approximately 17 hours 

o The system should be removed and replaced by a fresh one every 72 hours 

o The TDS is useful in patients with chronic, stable pain as well as patients who cannot 

swallow solid oral dosage forms (e.g., patients with head or neck cancer) 

o Counseling patients on proper use and disposal of the TDS is imperative 

Methadone Clinical Pearls 

o Useful and inexpensive agent for oncology patients 

o Thought to provide additional pain relief by antagonizing N‐methyl d‐aspartate (NMDA) 

receptors in the dorsal horn of the spinal cord 

o Analgesic duration (approximately 4‐8 hours) and drug half‐life (15‐30 hours) are very 

different 

o Drug accumulation may occur 

o Equianalgesic conversion is not linear, and very challenging 

o Doses should not be titrated faster than once every 5 days 

o Prolongs the QTc interval on the ECG (see the article by Krantz MJ et al in the pain 

management section of the reference list in this handout for suggested monitoring 

parameters) 

Opioid Rotation (this is a more complex and conservative example than that provided during the 

presentation, but it illustrates the point that conversion from one agent to another is not 

straightforward; see the article by Portenoy (Lancet) in the pain management section of the 

reference list in this handout) 

o Step 1 

Select agent based on previous patient experience, availability, cost, etc 

Calculate equianalgesic dose 

For opioids other than methadone and fentanyl, identify a 25‐50% dose reduction 

window lower than the calculated dose 

For methadone, the dose reduction window is 75‐90%, rarely converting to 

methadone at doses higher than 100 mg per day 

For transdermal fentanyl, use the package insert 

Select dose closer to the lower bound (25%) or upper bound (50%) based on how 

applicable the conversion is to the regimen and patient 

Upper bound if patient is on a fairly high dose, is not white, elderly or 

medically frail 

Lower bound if otherwise and especially if switching route of administration 

   

©2016 American Society of Health-System Pharmacists, Inc. All rights reserved.

____________________________________________________________________________________________________________ 17

Page 18: 2016 Pharmacotherapy Specialty Examination Review Course ...elearning.ashp.org/Files/Org/c52850f8e2e14ca8b14b... · 2016 Pharmacotherapy Specialty Examination Review Course: Complex

  

o Step 2 

On the basis of assessment of pain severity and other medical/psychosocial patient 

characteristics, increase or decrease the calculated dose by 15‐30% to improve the 

likelihood that the initial dose will be effective and/or prevent adverse 

effects/withdrawal 

Assess response and titrate as necessary to optimize outcome 

ASCO released guidelines for prevention and management of chemotherapy‐induced peripheral 

neuropathy in survivors of adult cancer (2014).  There is a lack of evidence for preventative therapy.  

The guidelines make a moderate recommendation for the use of duloxetine based on a randomized 

controlled trial for 5 weeks duration that resulted in a decrease in average pain intensity score in the 

duloxetine arm of 1.06 (95% CI, 0.72‐1.40) vs. 0.34 (95% CI, 0.01‐0.66) in the placebo arm (P = .003; 

effect size, 0.513).   

 

Breast Cancer Pearls 

 

Most common malignancy in women (approximately 231, 840 new cases expected in 2015), and 

second leading cancer killer behind lung cancer. Mortality from breast cancer appears to be 

declining.   

Risk factors 

o Gender (please remember that breast cancer can occur in men) 

o Advanced age 

o Genetics (BRAC1/2, others) 

o Family history, personal history 

o Race (Caucasian women have slightly higher incidence, but African‐American women have a 

higher mortality; Asian, Hispanic, and Native‐American women have a lower risk) 

o Dense breast tissue 

o Benign breast conditions 

o Lobular carcinoma in situ (LCIS) 

o Menarche at age < 12 and menopause at age > 55 (more hormonal exposure) 

o Diethylstilbestrol exposure 

o Increased hormone exposure 

Nulliparity 

Lack of breast feeding 

Oral contraceptives 

Hormone‐replacement therapy 

o Alcohol use 

o Obesity 

o Lack of physical activity 

Guidelines on breast cancer screening in average risk women have recently been updated, but there 

is no clear consensus on the age for initiation of screening or the screening frequency 

Cancers may arise from the ducts or lobules (85‐90% of invasive are ductal) 

Test all tumors for estrogen receptor (ER), progesterone receptor (PR) and HER2 status 

©2016 American Society of Health-System Pharmacists, Inc. All rights reserved.

____________________________________________________________________________________________________________ 18

Page 19: 2016 Pharmacotherapy Specialty Examination Review Course ...elearning.ashp.org/Files/Org/c52850f8e2e14ca8b14b... · 2016 Pharmacotherapy Specialty Examination Review Course: Complex

  

Treatment includes local therapy with surgery and/or radiation therapy and the treatment of 

systemic disease with chemotherapy, endocrine therapy, biologic therapy, or combinations of these 

modalities. 

Factors that influence treatment include: 

o Tumor histology 

o Clinical and pathologic characteristics of the tumor 

o Axillary node status 

o Tumor hormone receptor content 

o Tumor HER2 status 

o Multi‐gene testing 

o Patient factors: comorbid conditions, age, and menopausal status 

Patients stratified for treatment: 

o Pure noninvasive carcinomas: LCIS and ductal carcinoma in situ (DCIS) (Clinical Stage 0) 

o Operable, local‐regional invasive carcinomas with or without associated noninvasive 

carcinoma (Clinical Stage I, Stage IIA, Stage IIB, and some Stage IIIA) 

o Inoperable local‐regional invasive carcinoma with or without associated noninvasive 

carcinoma (Clinical Stage IIIB, Stage IIIC, and some Stage IIIA) 

o Metastatic (Clinical Stage IV) or recurrent carcinoma 

Survival Data (Based on 2001‐2002 diagnoses, American Cancer Society)  

 

Stage  5‐Year Overall Survival (%) 

0  93 

I  88 

IIA  81 

IIB  74 

IIIA  67 

IIIB  41 

IIIC  49 

IV  15  

Treatment 

o LCIS 

Surveillance 

Risk reduction counseling 

o DCIS 

Lumpectomy + radiation (Category 1, but no survival advantage) 

Mastectomy +/‐ reconstruction 

Lumpectomy alone with observation 

Consider tamoxifen for ER positive 

o Stage I, IIA, IIB, some IIIA 

Mastectomy with axillary lymph node dissection OR breast‐conserving therapy with 

lumpectomy, axillary dissection, and whole breast irradiation (Stages I, II) 

©2016 American Society of Health-System Pharmacists, Inc. All rights reserved.

____________________________________________________________________________________________________________ 19

Page 20: 2016 Pharmacotherapy Specialty Examination Review Course ...elearning.ashp.org/Files/Org/c52850f8e2e14ca8b14b... · 2016 Pharmacotherapy Specialty Examination Review Course: Complex

  

Consider neoadjuvant therapy for large IIA, IIB and T3N1M0 tumors when patients 

want to undergo breast‐conserving therapy (sentinel lymph node dissection should 

be completed before neoadjuvant therapy) 

Neoadjuvant pertuzumab and trastuzumab‐based regimens should be 

added to chemotherapy for patients with HER2 positive tumors 

Individualized endocrine therapy (if ER/PR positive), chemotherapy 

Node status 

4 positive – high risk for recurrence  prophylactic chest wall/regional 

lymph node radiation after mastectomy and chemotherapy (can be given 

with endocrine therapy, HER2‐targeted therapy) 

1‐3 positive – controversial; radiation recommended for large (>5 cm) 

tumors and/or positive margins 

Node negative – radiation for large tumors and/or positive margins 

Adjuvant Therapy 

Data for chemotherapy not definitive over age 70 

Online decision making tool (www.adjuvantonline.com) can be used to 

assess need for adjuvant therapy (endocrine, chemotherapy) 

Gene expression profile (DNA microarray technology) may be used to guide 

therapy; trials ongoing 

Not needed for small tumors <0.5 cm with no nodal involvement 

Lymph node negative, hormone receptor negative tumor > 1 cm  

chemotherapy 

Lymph node negative, hormone receptor positive tumor > 1 cm  

chemotherapy and endocrine therapy 

Lymph node positive  chemotherapy (and endocrine therapy if hormone 

receptor positive) 

Endocrine receptor positive 

o Adjuvant therapy should follow chemotherapy 

Postmenopausal  tamoxifen for 4.5‐6 years or aromatase 

inhibitor(AI) for 5 years.  Consider tamoxifen or aromatase inhibitor 

for an additional 5 years (up to 10 years of therapy). Women with a contraindication to or who are intolerant to or 

decline therapy with aromatase inhibitors: tamoxifen for 5 years or 

consider tamoxifen for up to 10 years 

o Premenopausal  aromatase inhibitor plus ovarian ablation or 

suppression or tamoxifen +/‐ ovarian ablation or suppression X 5 

years, consider an additional 5 years of tamoxifen or aromatase 

inhibitor (if initially treated with tamoxifen) 

Patients who become postmenopausal can complete 5 

years of AI therapy, then consider an additional 5 years of 

tamoxifen 

Chemotherapy – preferred regimens all evaluated in Phase III trials (NCCN® 

designates preferred vs. other based on efficacy and toxicity) 

©2016 American Society of Health-System Pharmacists, Inc. All rights reserved.

____________________________________________________________________________________________________________ 20

Page 21: 2016 Pharmacotherapy Specialty Examination Review Course ...elearning.ashp.org/Files/Org/c52850f8e2e14ca8b14b... · 2016 Pharmacotherapy Specialty Examination Review Course: Complex

  

o Dose‐dense AC followed by sequential paclitaxel every 2 weeks 

o Dose‐dense AC followed by weekly paclitaxel 

o TC (docetaxel and cyclophosphamide) 

o Many “other” options 

Adjuvant trastuzumab therapy for HER2‐positive disease X 1 year for tumors 

> 1 cm (+/‐ pertuzumab) 

o AC followed by paclitaxel with trastuzumab for 1 year starting with 

the first dose of paclitaxel 

o Consider docetaxel and carboplatin (TCH) for patients with risk 

factors for cardiac toxicity 

o Inoperable Stage III Invasive Tumors 

Neoadjuvant anthracycline‐based chemotherapy with or without a taxane 

Preoperative trastuzumab if HER2‐positive 

Total mastectomy with axillary lymph node dissection with or without delayed 

breast reconstruction OR lumpectomy and axillary dissection 

No matter what surgery, patients are at high risk for recurrence and therefore 

should receive adjuvant radiation 

Completion of chemotherapy course if not all given preoperatively 

Endocrine therapy and/or trastuzumab +/‐ pertuzumab as above for those with 

hormone receptor‐positive disease and/or HER2‐positive disease, respectively 

o Metastatic (Stage IV) or Recurrent  

Local recurrence 

Previous mastectomy only  salvage surgery if possible and radiation 

Previous mastectomy + radiation  limited duration systemic therapy 

Previous breast‐conserving therapy  mastectomy, axillary lymph node 

dissection if not done prior 

Systemic therapy using the principles of adjuvant therapy 

Systemic disease 

Endocrine therapy preferred due to improved toxicity profile in the 

palliative setting 

Patients are stratified by presence of bone metastases then hormone 

receptor and HER2 status 

Initiate bisphosphonate or RANK ligand therapy for those with bone disease 

o Monitoring includes serum calcium, phosphate, magnesium, renal 

function 

o Use calcium and vitamin D supplementation 

o Bone‐modifying therapies associated with osteonecrosis of the jaw 

(ONJ) 

Postmenopausal endocrine therapy options: anastrozole, letrozole, 

exemestane, tamoxifen, toremifene, fulvestrant, megestrol acetate, 

fluoxymesterone, ethinyl estradiol 

o The combination of everolimus and exemestane can be utilized in 

patients that have failed nonsteroidal aromatase inhibitors 

©2016 American Society of Health-System Pharmacists, Inc. All rights reserved.

____________________________________________________________________________________________________________ 21

Page 22: 2016 Pharmacotherapy Specialty Examination Review Course ...elearning.ashp.org/Files/Org/c52850f8e2e14ca8b14b... · 2016 Pharmacotherapy Specialty Examination Review Course: Complex

  

o The combination of palbociclib, a novel oral cyclin‐dependent kinase 

(CDK) inhibitor that blocks transition from the G1 to S phase of the 

cell cycle, can be used in combination with letrozole in 1st line 

therapy or with fulvestrant after progression on another endocrine 

therapy 

Premenopausal patients should receive ovarian suppression or ablation and 

follow postmenopausal options for treatment 

Chemotherapy 

o For hormone receptor negative tumors not localized to bone, 

symptomatic visceral metastases, refractory to endocrine therapy 

o NCCN® preferred single agents 

Anthracyclines – doxorubicin, liposomal doxorubicin 

Taxanes – paclitaxel, docetaxel, albumin‐bound paclitaxel 

Antimetabolites – capecitabine, gemcitabine 

Non‐taxane microtubule inhibitors – vinorelbine, eribulin 

o Preferred combination regimens 

CAF (cyclophosphamide, doxorubicin, fluorouracil) 

FEC (fluorouracil, epirubicin, cyclophosphamide) 

AC (doxorubicin, cyclophosphamide) 

EC (epirubicin, cyclophosphamide) 

CMF (cyclophosphamide, methotrexate, fluorouracil) 

Docetaxel and capecitabine 

GT (gemcitabine and paclitaxel) 

Gemcitabine and carboplatin 

Paclitaxel and bevacizumab (no OS benefit) 

o Failure to achieve a tumor response with 3 sequential 

chemotherapy regimens or ECOG performance status ≥ 3 is an 

indication for supportive care 

Add HER2‐targeted therapies for HER2‐positive disease 

o Trastuzumab + pertuzumab + docetaxel in patients who are 

treatment naïve (preferred, Category 1) (with paclitaxel, Category 

2A) 

o “Other 1st line options;” trastuzumab with nonanthracycline 

chemotherapy 

o Ado‐trastuzumab emtansine in patients that have previously 

received trastuzumab (preferred) 

o “Other” regimes for trastuzumab exposed patients include 

pertuzumab and trastuzumab (if no previous pertuzumab) +/‐ 

chemotherapy; capecitabine and lapatinib; lapatinib and letrozole 

o Do not use trastuzumab in combination with AC because of high risk 

for cardiotoxicity (27% is considered to be too high for the palliative 

setting) 

   

©2016 American Society of Health-System Pharmacists, Inc. All rights reserved.

____________________________________________________________________________________________________________ 22

Page 23: 2016 Pharmacotherapy Specialty Examination Review Course ...elearning.ashp.org/Files/Org/c52850f8e2e14ca8b14b... · 2016 Pharmacotherapy Specialty Examination Review Course: Complex

  

References and Recommended Readings  

 

Febrile Neutropenia 

 

1. Frefield AG, Bow EJ, Sepkowitz KA et al. Clinical practice guideline for the use of antimicrobial agents 

in neutropenic patients with cancer: 2010 update by the Infectious Disease Society of America. Clin 

Infect Dis. 2011; 52:e56‐93. 

http://www.uphs.upenn.edu/bugdrug/antibiotic_manual/idsaneutropenicfever2010.pdf (accessed 

2015 May 19). 

 

2. National Comprehensive Cancer Network. NCCN® clinical practice guidelines in oncology: prevention 

and treatment of cancer‐related infections. Version 2.2015. Available at: 

http://www.nccn.org/professionals/physician_gls/pdf/infections.pdf.  Referenced with permission 

from the NCCN Clinical Practice Guidelines in Oncology (NCCN Guidelines®) for Prevention and 

Treatment of Cancer‐Related Infections V.2.2015 © National Comprehensive Cancer Network, Inc 

2015. All rights reserved. Accessed [November 1, 2015]. 

 

3. Mermel LA, Allon M, Bouza E et al. Clinical practice guidelines for the diagnosis and management of 

intravascular catheter‐related infection: 2009 update by the Infectious Disease Society of America. 

Clin Infect Dis. 2009; 49:1‐45. http://www.ncbi.nlm.nih.gov/pubmed/19489710 (accessed 2012 May 

31). 

 

4. Pappas PG, Kauffman CA, Andes D et al. Clinical practice guidelines for the management of 

candidiasis: 2009 update by the Infectious Diseases Society of America. Clin Infect Dis. 2009; 48:503‐

35. http://www.ncbi.nlm.nih.gov/pubmed/19191635 (accessed 2014 May 21). 

 

5. Klastersky J, Paesmans M, Rubenstein EB et al. The Multinational Association for Supportive Care in 

Cancer risk index: a multinational scoring system for identifying low‐risk febrile neutropenic cancer 

patients. J Clin Oncol. 2000; 18:3038‐51. http://www.ncbi.nlm.nih.gov/pubmed/10944139 (accessed 

2014 May 21). 

 

6. Walsh TJ, Anaissie EJ, Denning DW et al. Treatment of aspergillosis: clinical practice guidelines of the 

Infectious Diseases Society of America. Clin Infect Dis. 2008; 46:327‐60. 

http://www.uphs.upenn.edu/bugdrug/antibiotic_manual/idsa‐aspergilloisrx‐2008guidelines.pdf 

(accessed 2014 May 21). 

 

7. Flowers CR, Seidenfeld J, Bow EJ et al. Antimicrobial prophylaxis and outpatient management of 

fever and neutropenia in adults treated for malignancy: American Society of Clinical Oncology 

clinical practice guideline. J Clin Oncol. 2013; 31:794‐810.  

http://jco.ascopubs.org/content/31/6/794.long  (accessed 2014 May 21) 

 

   

©2016 American Society of Health-System Pharmacists, Inc. All rights reserved.

____________________________________________________________________________________________________________ 23

Page 24: 2016 Pharmacotherapy Specialty Examination Review Course ...elearning.ashp.org/Files/Org/c52850f8e2e14ca8b14b... · 2016 Pharmacotherapy Specialty Examination Review Course: Complex

  

Anticoagulation 

 

1. Lee AYY, Levine MN, Baker RI et al. Low‐molecular‐weight heparin versus a coumarin for the 

prevention of recurrent venous thromboembolism in patients with cancer. N Engl J Med. 2003; 

349:146‐53. http://www.nejm.org/doi/full/10.1056/NEJMoa025313 (accessed 2014 May 21). 

 

2. Kearon C, Akl EA, Comerota AJ et al. Antithrombotic therapy for VTE disease: antithrombotic therapy 

and prevention of thrombosis, 9th Ed: American College of Chest Physicians evidence‐based clinical 

practice guidelines. Chest. 2012; 141(2 suppl):419S‐94S. 

http://www.ncbi.nlm.nih.gov/pubmed/22315268 (accessed 2014 May 21).   

 

3. National Comprehensive Cancer Network. NCCN® clinical practice guidelines in oncology: venous 

thromboembolic disease. Version 1.2015. Available at: 

http://www.nccn.org/professionals/physician_gls/pdf/vte.pdf.  Referenced with permission from 

the NCCN Clinical Practice Guidelines in Oncology (NCCN Guidelines®) for Venous Thromboembolic 

Disease V.1.2015 © National Comprehensive Cancer Network, Inc 2014. All rights reserved. 

Accessed [November 1 2015]. 

 

4. Lyman GH, Khorana AA, Kuderer NM et al. Venous thromboembolism prophylaxis and treatment in 

patients with cancer: American Society of Clinical Oncology clinical practice guideline update. J Clin 

Oncol.  2013; 31:2189‐204. http://jco.ascopubs.org/content/31/17/2189.long (accessed 2014 May 

21).  

 

5. Lee AYY, Kamphuisen PW, Meyer G et al.  Tinzaparin vs warfarin for treatment of acute venous 

thromboembolism in patients with active cancer: a randomized clinical trial.  JAMA. 2015; 314:677‐

86.  http://jama.jamanetwork.com/article.aspx?articleid=2428955&resultClick=3 (accessed 2016 

February 21). 

 

6. Kearon C, Akl EA, Ornelas J et al.  Antithrombotic therapy for VTE disease: CHEST guideline and 

expert panel report.  Chest.  2016; 149: 315‐52.  

http://journal.publications.chestnet.org/article.aspx?articleid=2479255 (accessed 2016 February 

21). 

 

Nausea and Vomiting 

 

1.   Hesketh PJ, Bohlke K, Lyman GH et al. Antiemetics: American Society of Clinical Oncology focused 

guideline update. J Clin Oncol. 2016; 34:381‐86. 

http://jco.ascopubs.org/content/early/2015/10/26/JCO.2015.64.3635.full  (accessed 2016 Feb 8)  

 

2.    Gralla RJ, Rolia F, Tonato M et al. MASCC/ESMO antiemetic guidelines 2013. Available at: 

http://www.mascc.org/antiemetic‐guidelines. 

 

©2016 American Society of Health-System Pharmacists, Inc. All rights reserved.

____________________________________________________________________________________________________________ 24

Page 25: 2016 Pharmacotherapy Specialty Examination Review Course ...elearning.ashp.org/Files/Org/c52850f8e2e14ca8b14b... · 2016 Pharmacotherapy Specialty Examination Review Course: Complex

  

3. National Comprehensive Cancer Network. NCCN® clinical practice guidelines in oncology: 

antiemesis. Version 2.2015. Available at: 

http://www.nccn.org/professionals/physician_gls/pdf/antiemesis.pdf.  Referenced with permission 

from the NCCN Clinical Practice Guidelines in Oncology (NCCN Guidelines®) for Antiemesis  V.2.2015 

© National Comprehensive Cancer Network, Inc 2015. All rights reserved. Accessed [February 8, 

2016]. 

 

4.    Hesketh PJ. Chemotherapy induced nausea and vomiting. N Engl J Med. 2008; 358:2482‐94.  

http://www.motilitysociety.org/clinician/manuscripts/drugNV_nejm.pdf (accessed 2014 May 21) 

 

Pain Management 

 

1. World Health Organization. Cancer pain relief: with a guide to opioid availability. 2nd ed. Geneva; 

1996. Available at http://whqlibdoc.who.int/publications/9241544821.pdf  

 

2. Portenoy RK. Treatment of cancer pain. Lancet. 2011; 377:2236‐47. 

http://www.thelancet.com/journals/lancet/article/PIIS0140‐6736(11)60236‐5/fulltext (accessed 

2014 May 21).  

 

3. Krantz MJ, Martin J, Stimmel B et al. QTc interval screening in methadone treatment. Ann Intern 

Med. 2009; 150:387‐95. http://www.ncbi.nlm.nih.gov/pubmed/19153406 (accessed 2014 May 21). 

 

4. National Comprehensive Cancer Network. NCCN® clinical practice guidelines in oncology: adult 

cancer pain. Version 2.2015. Available at: 

http://www.nccn.org/professionals/physician_gls/pdf/pain.pdf.  Referenced with permission from 

the NCCN Clinical Practice Guidelines in Oncology (NCCN Guidelines®) for Adult Cancer Pain 

V.2.2015 © National Comprehensive Cancer Network, Inc 2015. All rights reserved. Accessed 

[November 1, 2015]. 

 

5. Portenoy RK, Ahmend E.  Principles of opioid use in cancer pain.  J Clin Oncol.  2014;  32: 1662‐70.  

 

6. Hui D, Bruera E.  A personalized approach to assessing and managing pain in patients with cancer.  .  

J Clin Oncol.  2014;  32: 1640‐46.   

 

7. Hershman DL, Lacchetti C, Dworkin RH, et al. Prevention and management of chemotherapy‐

induced peripheral neuropathy in survivors of adult cancers: American Society of Clinical Oncology 

clinical practice guideline.  J Clin Oncol.  2014; 32:1941‐67.  

 

   

©2016 American Society of Health-System Pharmacists, Inc. All rights reserved.

____________________________________________________________________________________________________________ 25

Page 26: 2016 Pharmacotherapy Specialty Examination Review Course ...elearning.ashp.org/Files/Org/c52850f8e2e14ca8b14b... · 2016 Pharmacotherapy Specialty Examination Review Course: Complex

  

Breast Cancer 

 

1. National Comprehensive Cancer Network. NCCN® clinical practice guidelines in oncology: breast 

cancer. Version 1.2016. Available at: 

http://www.nccn.org/professionals/physician_gls/pdf/breast.pdf.  Referenced with permission from 

the NCCN Clinical Practice Guidelines in Oncology (NCCN Guidelines®) for Breast Cancer V.1.2016 © 

National Comprehensive Cancer Network, Inc 2016. All rights reserved. Accessed [February 28, 

2016]. 

 

2. Burnstein HJ, Temin S,  Anderson H et al. Adjuvant endocrine therapy for women with hormone 

receptor‐positive breast cancer: American Society of Clinical Oncology clinical practice guideline 

focused update.. J Clin Oncol. 2014; 32:2255‐69. 

 

3. Ellis GK, Livingston, RB, Gralow JR et al. Dose‐dense anthracycline‐based chemotherapy for node‐

positive breast cancer. J Clin Oncol. 2002; 20:3637‐43. 

http://jco.ascopubs.org/content/20/17/3637.long (accessed 2014 May 21). 

 

4. Romond EH, Perez EA, Bryant J et al. Trastuzumab plus adjuvant chemotherapy for operable HER2‐

positive breast cancer. N Engl J Med. 2005; 353:1673‐84. 

http://www.nejm.org/doi/full/10.1056/NEJMoa052122 (accessed 2014 May 21). 

 

5. Irvin W Jr, Muss HB, Mayer DK. Symptom management in metastatic breast cancer. Oncologist. 

2011; 16:1203‐14. http://www.ncbi.nlm.nih.gov/pubmed/21880861 (accessed 2014 May 21). 

 

6. Swain SM, Basegla J, Kim SJ, et al.  Pertuzumab, trastuzumab, and docetaxel in HER2 positive 

metastatic breast cancer.  N Engl J Med.  2015; 372: 724‐34.  

 

7. Turner NC, Ro J, Andre F, et al.  Palbociclib in hormone‐receptor‐positive advanced breast cancer.  N 

Engl J Med.  2015; 373:209‐19.   

 

8. Giordano SH, Temin S, Kirshner JJ, et al.  Systemic therapy for patients with advanced human 

epidermal growth factor receptor 2‐positive breast cancer: American Society of Clinical Oncology 

practice guideline.  J Clin Oncol.  2014; 32: 2078‐99. 

 

9. Partridge AH, Rumble RB, Carey AL, et al.  Chemotherapy and targeted therapy for women with 

human epidermal growth factor receptor‐negative (or unknown) advanced breast cancer: American 

Society of Clinical Oncology practice guideline.  J Clin Oncol.  2014; 32: 3307‐29. 

 

10. Oeffinger KC, Fontham ETH, Etzioni R et al.  Breast cancer screening for women at average risk: 2015 

guideline update from the American Cancer Society.  JAMA.  2015; 314: 1599‐1614.   

 

11. Siu AL, US Preventive Services Task Force.  Screening for breast cancer: US Preventive Services Task 

Force recommendation statement.  Ann Intern Med.  2016; 164: 279‐96.   

 

©2016 American Society of Health-System Pharmacists, Inc. All rights reserved.

____________________________________________________________________________________________________________ 26

Page 27: 2016 Pharmacotherapy Specialty Examination Review Course ...elearning.ashp.org/Files/Org/c52850f8e2e14ca8b14b... · 2016 Pharmacotherapy Specialty Examination Review Course: Complex

  

12. National Comprehensive Cancer Network. NCCN® clinical practice guidelines in oncology: breast 

cancer screening and diagnosis. Version 1.2015. Available at: 

http://www.nccn.org/professionals/physician_gls/pdf/breast‐screening.pdf.  Referenced with 

permission from the NCCN Clinical Practice Guidelines in Oncology (NCCN Guidelines®) for Breast 

Cancer Screening and Diagnosis V.1.2015 © National Comprehensive Cancer Network, Inc 2015. All 

rights reserved. Accessed [February 21, 2016]. 

 

Safety in Cancer Patients 

 

1. Verma S, Madarnas Y, Shedev S et al. Patient adherence to aromatase inhibitor treatment in the 

adjuvant setting. Current Oncology. 2011; 18(Suppl 1):S3‐9.  

http://www.ncbi.nlm.nih.gov/pmc/articles/PMC3119895/ (accessed 2014 May 21) 

 

2. Holbrook AM, Pereira JA, Labiris R et al. Systemic overview of warfarin and its drug and food 

interactions. Arch Intern Med. 2005; 165:1095‐106. 

http://www.ncbi.nlm.nih.gov/pubmed/15911722 (accessed 2014 May 21). 

 

3. Schulmeister L. Extravasation management: clinical update. Semin Oncol Nurs. 2011; 27:82‐90. 

http://www.ncbi.nlm.nih.gov/pubmed/21255716 (accessed 2014 May 21). 

 

4. Senkus E, Jassem J. Cardiovascular effects of systemic cancer treatment. Cancer Treat Rev. 2011; 

37:300‐11. http://www.ncbi.nlm.nih.gov/pubmed/21126826 (accessed 2014 May 21). 

 

Patient Resources 

 

1. Jefford M, Tattersall MHN. Informing and involving cancer patients in their own care. Lancet Oncol. 

2002; 3:629‐37. http://www.thelancet.com/journals/lanonc/article/PIIS1470‐2045(02)00877‐

X/abstract (accessed 2014 May 21). 

 

2. NCCN® Guidelines for Patients (http://www.nccn.org/patients/default.asp).  Referenced with 

permission from the NCCN Clinical Practice Guidelines in Oncology (NCCN Guidelines®) for Patients 

© National Comprehensive Cancer Network, Inc 2016. All rights reserved. Accessed [February 28, 

2016]. 

 

3. ASCO Website for Patients (http://www.cancer.net) 

 

4. American Cancer Society (http://www.cancer.org/) 

 

5. LIVESTRONG Foundation (http://www.livestrong.org/) 

 

6. Clinical Trials (www.clinicaltrials.gov) 

©2016 American Society of Health-System Pharmacists, Inc. All rights reserved.

____________________________________________________________________________________________________________ 27

Page 28: 2016 Pharmacotherapy Specialty Examination Review Course ...elearning.ashp.org/Files/Org/c52850f8e2e14ca8b14b... · 2016 Pharmacotherapy Specialty Examination Review Course: Complex

Complex Breast Cancer Case

Helen M. Marshall, Pharm.D., BCPS, BCOPClinical Pharmacist, Hematopoietic Stem Cell Transplant

Seattle Cancer Care Alliance ‐ University of Washington Medical Center

Clinical Assistant Professor

University of Washington School of Pharmacy

Seattle, Washington

• I have nothing to disclose related to the content of this presentation.

Disclosure

Learning Objectives

• Select the appropriate treatment and monitoring of a complex patient‐case with multiple conditions, including breast cancer, febrile neutropenia, and pulmonary embolism.

• Compare and contrast current breast cancer screening recommendations. 

• Develop a plan to manage pain and nausea/vomiting in a patient with cancer.

Learning Objectives

• Determine how to manage drug‐drug and drug‐disease interactions in a cancer patient.

• Discuss safety issues in this population.

• Identify and recommend appropriate resource organizations/groups to assist a specific patient.

Our Patient (HPI)• LK is a 54 year old female who presents to a multidisciplinary breast cancer clinic at your comprehensive cancer center to establish a treatment plan for her newly diagnosed right breast cancer

• Right breast lump discovered by LK’s husband 

• Work up revealed localized breast cancer

• LK reports no associated symptoms

• LK has no primary care physician and reports no previous health conditions

She has not had a menstrual cycle in the last 3 years

LK Continued

• FH: Father: MI s/p CABG X 2 at age 55; Mother has a history of stage I breast cancer s/p mastectomy with reconstruction and no recurrence; Brother has MS 

• SH: Lives at home with husband, 2 daughters in college; completed high school in China; speaks Mandarin;  former smoker (1 pack per day from age 18‐30), no alcohol use

• NKDA

• Height 167 cm, Weight 78 kg, BSA 1.9 

©2016 American Society of Health-System Pharmacists, Inc. All rights reserved.

____________________________________________________________________________________________________________ 28

Page 29: 2016 Pharmacotherapy Specialty Examination Review Course ...elearning.ashp.org/Files/Org/c52850f8e2e14ca8b14b... · 2016 Pharmacotherapy Specialty Examination Review Course: Complex

LK Continued

• Breast Cancer, Stage IIB

Diagnosed March 2016 in her right breast

Stage IIB (T2 N1 M0)

ER and PR positive

HER2 negative

Postmenopausal (menopause at age 51)

Question 1: Should LK have previously undergone screening with mammography prior to her diagnosis?

A. Yes, annually starting at age 20

B. Yes, due to her family history

C. No, because LK is under age 55

D. No, due to her family history

Breast Cancer Screening with Mammography• BENEFITS

Decrease in breast cancer mortality

—For women age 40‐74, screening is associated with a 15‐20% relative risk reduction in mortality

—Based on 25 year follow up of the Canadian National Breast Cancer Screening Study (RCT), there is uncertainty about the magnitude of benefit (2014)

• RISKS

Overdiagnosis and resulting treatment of insignificant cancers

False positives with additional testing and anxiety

False negatives with a false sense of security and potential delay in diagnosis

Radiation‐induced breast cancer Nelson HD, et al.  Ann Intern Med.  2009;151:727‐37.Miller AB, et al.  BMJ. 2014;348:g366.

National Cancer Institute PDQ® Breast Cancer Screening.  Bethesda, MD: National Cancer Institute.  Date last modified 01/08/2016.  Available at: http://www.cancer.gov/types/breast/hp/breast‐screening‐pdq.  Accessed 01/31/2016.

American Cancer Society (ACS) Key Recommendations for Women at Average Risk• Regular screening mammography starting at age 45 years (strong)

• Women aged 45 to 54 years should be screened annually (qualified)

• Women 55 years and older should transition to biennial screening or have the opportunity to continue screening annually (qualified)

• Women should have the opportunity to begin annual screening between the ages of 40 and 44 years (qualified)

• Continue screening mammography as long as overall health is good and they have a life expectancy of 10 years or longer (qualified)

• ACS does not recommend clinical breast examination for screening at any age (qualified)

• Last update 2003

Oeffinger KC et al. JAMA. 2015; 314:1599‐1614. 

US Preventive Services Task Force (USPSTF) Recommendation (for Average Risk Women)• Biennial screening mammography for women aged 50 to 74 years (B recommendation)

• The decision to start screening mammography in women prior to age 50 years should be an individual one. Women who place a higher value on the potential benefit than the potential harms may choose to begin biennial screening between the ages of 40 and 49 years (C recommendation)

• Current evidence is insufficient to assess the balance of benefits and harms of screening mammography in women aged 75 years or older (I statement)

• Last update 2009

Siu AL.  Ann Intern Med.  2016; 164:279‐96.  

National Comprehensive Cancer Network (NCCN) Guidelines® Breast Cancer Screening and Diagnosis (Average Risk Women)

• Age ≥ 40 years

Annual clinical breast exam

Annual screening mammogram (Category 1)

Breast awareness

• Age ≥ 25 but < 40 years

Clinical breast exam every 1 to 3 years

Breast awareness

Referenced with permission from the NCCN Clinical Practice Guidelines in Oncology (NCCN Guidelines®) for Breast Cancer Screening and Diagnosis  V.1.2015 © National Comprehensive Cancer Network, Inc. 2015. All rights reserved. Accessed [February 21, 2016].

©2016 American Society of Health-System Pharmacists, Inc. All rights reserved.

____________________________________________________________________________________________________________ 29

Page 30: 2016 Pharmacotherapy Specialty Examination Review Course ...elearning.ashp.org/Files/Org/c52850f8e2e14ca8b14b... · 2016 Pharmacotherapy Specialty Examination Review Course: Complex

LK ‐ Planned Breast Cancer Treatment 

• Surgery

Mastectomy with axillary lymph node dissection

4 positive lymph nodes

• Radiation

Chest wall and regional lymph nodes

Planned Breast Cancer Treatment• Chemotherapy

Dose‐dense AC (Adriamycin[doxorubicin] and cyclophosphamide)

—Doxorubicin 60 mg/m2 IV day 1

—Cyclophosphamide 600 mg/m2 IV day 1

—Every 14 days X 4 cycles

—Dose‐dense AC requires growth factor support

Followed by paclitaxel

—Paclitaxel 80 mg/m2 IV day 1 weekly X 12 cycles

• Prevention of nausea and vomiting

Palonosetron 0.25 mg IV pre‐chemotherapy day 1

Dexamethasone 10 mg IV pre‐chemotherapy day 1

Question 2: Which of the following should be monitored for our patient on this chemotherapy regimen? 

A. CBC, liver function tests, renal function, and neurologic function

B. CBC, liver function tests, renal function, and signs and symptoms of hemorrhagic cystitis

C. CBC, liver function tests, and cardiac function

D. CBC, liver function tests, and signs and symptoms tumor lysis syndrome

Evaluating and Monitoring Chemotherapy 

• Patient

• Regimen

• Organ function

• Numbers

• Toxicity

Adverse effects

Supportive care

• Drug‐drug interactions

Question 3:  What is the appropriate antiemetic regimen for LK while receiving treatment with dose dense AC? 

A. Ondansetron 8 mg IV and dexamethasone 12 mg IV day 1 followed by dexamethasone 4 mg PO BID days 2‐4

B. Fosaprepitant 150 mg IV, ondansetron 8 mg IV and dexamethasone 12 mg IV day 1 followed by dexamethasone 4 mg PO BID days 2‐4

C. Fosaprepitant 150 mg IV, ondansetron 8 mg IV and dexamethasone 12 mg IV day 1 

D. Palonosetron 0.25 mg IV and dexamethasone 10 mg IV day 1 followed by dexamethasone 4 mg PO BID days 2‐4

Chemotherapy Induced Nausea and Vomiting (CINV)• Acute Emesis within the first 24 hours after treatment

• Delayed Emesis occurring at least 24 hours after treatment

• Neurotransmitters dopamine, 5‐HT, and substance P play a role in CINV

• Prevention and treatment governed by multiple sets of guidelines MASCC ASCO NCCN®

Hesketh PJ. N Engl J Med. 2008; 358:2482‐94.  

©2016 American Society of Health-System Pharmacists, Inc. All rights reserved.

____________________________________________________________________________________________________________ 30

Page 31: 2016 Pharmacotherapy Specialty Examination Review Course ...elearning.ashp.org/Files/Org/c52850f8e2e14ca8b14b... · 2016 Pharmacotherapy Specialty Examination Review Course: Complex

CINV Risk Factors

• Young age

• Female sex

• Expectation of severe nausea

• Chemotherapy

Dose 

Emetogenicity

• History of alcohol consumption (inverse)

Hesketh PJ. N Engl J Med. 2008; 358:2482‐94.

Highly Emetogenic Chemotherapy Agents

MASCC NCCN® ASCO

AGENTS CisplatinMechlorethamineStreptozocinCyclophosphamide > 1500 mg/m2

CarmustineDacarbazine

AC (doxorubicin or epirubicin with cyclophosphamide)Carmustine > 250 mg/m2

CisplatinCyclophosphamide > 1500 mg/m2DacarbazineDoxorubicin ≥ 60mg/m2

Epirubicin . 90 mg/m2

Ifosfamide ≥ 2 g/m2/doseMechlorethamineStreptozocin

CarmustineCisplatinCyclophosphamide ≥ 1500mg/m2

DacarbazineDactinomycinMechlorethamineStreptozocin

Specifics on the AC Regimen

Lists Anthracycline + Cyclophosphamide as its own category and is considered a highly emetogenic regimen

See above Asterisks anthracyclines (moderate level) and  states highly emetogenic when combined with cyclophosphamide

Highly Emetogenic is defined as likely to cause N/V in >90% of patients

Referenced with permission from the NCCN Clinical Practice Guidelines in Oncology (NCCN Guidelines®) for Antiemesis  V.2.2015 © National Comprehensive Cancer Network, Inc. 2015. All rights reserved. Accessed [November 1, 2015].

Basch E et al. J Clin Oncol. 2011; 29:4189‐98.Gralla RJ et al. MASCC/ESMO Antiemetic Guidelines. 2013.

Prevention of CINV (Highly Emetogenic)

MASCC NCCN® ASCO

Day 1 of Chemo

NK1 Antagonist Fosaprepitant 150 mg IVAprepitant 125 mg PO

Fosaprepitant 150 mg IV*Aprepitant 125 mg PONetupitant 300mg/Palonosetron 0.5mg PORolapitant 180 mg PO**

Fosaprepitant 150 mg IVAprepitant 125 mg PONetupitant 300mg/Palonosetron 0.5mg PO

5HT3 Antagonist See next  slide Palonosetron 0.25mg IV (preferred) See next  slidePalonosetron preferred

Corticosteroid Dexamethasone 12 mg  IV/PO

Dexamethasone 12 mg IV/PODexamethasone 20 mg IV/PO**

Dexamethasone 12 mg IV/PO

Subsequent Days

NK1 Antagonist Aprepitant 80 mg Days 2 and 3

Aprepitant 80 mg Days 2 and 3 Aprepitant 80 mg Days 2 and 3

Corticosteroid Dexamethasone 8 mg  PO  daily for 3‐4 days

Dexamethasone 8 mg PO Day 2 then 8 mg po BID Days 3‐4*Dexamethasone 8 mg IV/PO Days 2‐4

Dexamethasone 8 mg IV/PO Days 2‐3 or 2‐4

Additional Recommendations

+/‐ 0.5‐2 mg lorazepam Q 6 hours+/‐ H2 blocker or PPI

Notes Olanzapine based regimen also available

5HT3 Antagonist Options• In all three guidelines

• Dolasetron 100 mg PO ONLY (QT prolongation)

• Palonosetron 0.25 mg IV

• Granisetron 0.01mg/kg IV (1mg), 2 mg PO

• In ASCO and MASCC

• Palonosetron 0.5 mg PO

• Ondansetron 8 mg IV or 0.15mg/kg IV

• In NCCN®

• Ondansetron 16‐24 mg PO or 8‐16 mg IV

• Granisetron 1 mg po BID or transdermal patch 3.1mg/24 hour applied 24‐48 hours pre chemo, max 7 days

• In ASCO

• Ondansetron 8 mg PO BID

• In MASCC• Ondanseton 16 mg PO (8 mg po BID in randomized studies)

Breakthrough N/V Management• Limited prospective data

• Lack of consensus despite guidelines

• Treatment is largely empiric

• Agents frequently used Benzodiazepines

Phenothiazines

• Consider changing original preventative regimen

• NCCN® states the principle is to add an additional agent from a different drug class to the current regimen

Hesketh PJ. N Engl J Med. 2008; 358:2482‐94.  Referenced with permission from the NCCN Clinical Practice Guidelines in Oncology (NCCN Guidelines®) for Antiemesis  V.2.2015 © National Comprehensive Cancer Network, Inc 2015. All rights reserved. Accessed [February 8, 

2016].

Additional Agents for Breakthrough N/V• Phenothiazines

Prochlorperazine* 10 mg PO every 6 hours

Promethazine* 12.5‐25 mg PO (IV if central line only) every 4 hours

• Others

Haloperidol* 0.5‐2 mg PO/IV every 4‐6 hours

Metoclopramide* 10‐40 mg PO/IV every 4 to 6 hours

Scopolamine transdermal patch every 72 hours

• Atypical Antipsychotics

Olanzapine 10 mg po daily X 3 days

• Benzodiazepine

Lorazepam 0.5 – 2 mg PO/IV every 4‐6 hours

• Cannabinoid

Dronabinol 5‐10 mg PO every 3 or 6 hours

Nabilone 1‐2 mg PO BID

Referenced with permission from the NCCN Clinical Practice Guidelines in Oncology (NCCN Guidelines®) for Antiemesis  V.2.2015 © National Comprehensive Cancer Network, Inc 2015. All rights reserved. Accessed 

[February 8, 2016].

* Monitor for dystonic reactions; treat with diphenhydramine

©2016 American Society of Health-System Pharmacists, Inc. All rights reserved.

____________________________________________________________________________________________________________ 31

Page 32: 2016 Pharmacotherapy Specialty Examination Review Course ...elearning.ashp.org/Files/Org/c52850f8e2e14ca8b14b... · 2016 Pharmacotherapy Specialty Examination Review Course: Complex

LK’s Medications

• Chemo and premedication

• Ondansetron 8 mg po every 8 hours prn nausea and vomiting (N/V)

• Prochlorperazine 10 mg po every 6 hours prn N/V

• Lorazepam 0.5‐1 mg po every 6 hours prn N/V, anxiety

Interval History• 7 days after Cycle 3 of AC chemotherapy, LK calls into clinic with fever 101.2°F

• She reports sweats and chills, cough, and feeling poorly

• She asks to take acetaminophen, but her nurse recommends she come into clinic for evaluation

2 sets of blood cultures drawn (one from port, one peripherally), urine cultures obtained

Labs

—BMP: Na 129 mEq/L, K 3.3 mEq/L, Cl 100 mEq/L, CO2 32 mEq/L, BUN 25 mg/dL, SCr 1.2 mg/dL, Glucose 95 mg/dL;LFTs WNL

—CBC: WBC 0.9 thou/microL, Hgb 10.8 g/dL, Hct 27%, Plts 79 thou/microL, ANC 0.38 thou/microL

Vitals: RR 23, BP 87/62, HR 122, Temp 101.4°F 

Question 4:  Which of the following should be initiated for empiric therapy for febrile neutropenia for LK?

A. Ciprofloxacin 500 mg IV every 8 hours

B. Ciprofloxacin 500 mg PO every 8 hours and clindamycin 450 mg PO every 8 hours 

C. Ciprofloxacin 500 mg PO every 8 hours and amoxicillin/clavulanate500 mg PO every 8 hours

D. Imipenem 500 mg IV every 6 hours

Febrile Neutropenia (FN)• Definition

ANC <0.5 X 109/L  or an ANC that is expected to decrease to <0.5 X 109/L during the next 48 hours and 

A single oral temperature >101°F (38.3°C) or >100.4° F (>38°C) for at least 1 hour

• Common source of cancer‐related morbidity and mortality

• Fever is only reliable indicator of infection Beware of masking fever with acetaminophen (APAP)/NSAIDs

• Primary sites of infection GI tract Lungs, sinuses Skin

Freifeld AG et al. Clin Infect Dis. 2011; 52:ee56‐93. Referenced with permission from the NCCN Clinical Practice Guidelines in Oncology (NCCN 

Guidelines®) for Prevention and Treatment of Cancer‐Related Infections V.2.2015 © National Comprehensive Cancer Network, Inc 2015. All rights reserved. Accessed [November 1, 2015 ].

Risk Factors for FN

• Age

• Performance status

• Chemotherapy dose intensity

• Nutritional status

• Low baseline blood counts

Lyman GH et al. Oncologist. 2005; 10:427‐37. 

Treatment of FN

• Immediate 

• Utilize bactericidal agents

• Design guidelines based on your antibiogram and antimicrobial cost

• Utilize the MASCC, IDSA, and NCCN® guidelines to guide therapy

Referenced with permission from the NCCN Clinical Practice Guidelines in Oncology (NCCN Guidelines®) for Prevention and Treatment of Cancer‐Related Infections V.2.2015© National Comprehensive Cancer Network, Inc 

2015. All rights reserved. Accessed [November 1, 2015].Freifeld AG et al. Clin Infect Dis. 2011; 52:e56‐93.Klastersky J et al. J Clin Oncol. 2000; 18:3038‐51.

©2016 American Society of Health-System Pharmacists, Inc. All rights reserved.

____________________________________________________________________________________________________________ 32

Page 33: 2016 Pharmacotherapy Specialty Examination Review Course ...elearning.ashp.org/Files/Org/c52850f8e2e14ca8b14b... · 2016 Pharmacotherapy Specialty Examination Review Course: Complex

Guideline Comparison: FN Risk Stratification MASCC Scoring Index Characteristic Score NCCN® Low Risk

Extent of Illness No Symptoms 5

Mild Symptoms 5

Moderate Symptoms 3

SBP ≥ 90 mm Hg 5

No COPD 4 No acute comorbid illness

Score ≥ 21  indicates low risk

Solid tumor (or heme malignancy w/o previous fungal infection)

4Anticipated short duration of neutropenia (<0.1 X109/L < 7 days)

No dehydration 3 No hepatic or renal insufficiency

Outpatient at fever onset 3 Outpatient at fever onset

Age < 60 yr 2 ECOG Performance Status 0‐1

Referenced with permission from the NCCN Clinical Practice Guidelines in Oncology (NCCN Guidelines®) for Prevention and Treatment of Cancer‐Related Infections V.2.2015 © National Comprehensive Cancer Network, Inc 2015. All rights reserved. Accessed [November 1, 2015].

Klastersky J et al. J Clin Oncol. 2000; 18:3038‐51. Freifeld AG et al. Clin Infect Dis. 2011; 52:e56‐93.

Guideline Comparison: FN Risk Stratification MASCC Scoring Index Characteristic Score NCCN® Low Risk

Extent of Illness No Symptoms 5

Mild Symptoms 5

Moderate Symptoms 3

SBP ≥ 90 mm Hg 5

No COPD 4 No acute comorbid illness

Score ≥ 21  indicates low risk

Solid tumor (or heme malignancy w/o previous fungal infection)

4Anticipated short duration of neutropenia (<0.1 X109/L < 7 days)

No dehydration 3 No hepatic or renal insufficiency

Outpatient at fever onset 3 Outpatient at fever onset

Age < 60 yr 2 ECOG Performance Status 0‐1

Referenced with permission from the NCCN Clinical Practice Guidelines in Oncology (NCCN Guidelines®) for Prevention and Treatment of Cancer‐Related Infections V.2.2015 © National Comprehensive Cancer Network, Inc 2015. All rights reserved. Accessed [November 1, 2015].

Klastersky J et al. J Clin Oncol. 2000; 18:3038‐51. Freifeld AG et al. Clin Infect Dis. 2011; 52:e56‐93.

ASCO Guidelines: Antimicrobial Prophylaxis & Outpatient Management of Fever and Neutropenia

PROPHYLAXIS OUTPATIENT FN• Only use antibacterial and antifungal 

prophylaxis if neutrophils are expected to remain <100/µL for > 7 days (unless other factors increase risk for complications/mortality)

Oral fluoroquinolone

Oral triazole

• Interventions such as neutropenic diet, surgical/respiratory masks, supplements are not recommended (no data)

• Assess risk for medical complications in patients with FN

MASCC score (≥21)

Talcott’s rules (Talcott group 4)

No additional risk factors

• Oral fluoroquinolone plus amoxicillin/clavulanate (clindamycin for PCN allergic)

Unless FQ prophylaxis used prior to fever development

Give within 1 hour of triage

• Monitor patient for 4 hours to determine suitability of outpatient management

In 2014, NCCN® added moxifloxacin as a Category 1 recommendation for low risk FN treatment. Note moxifloxacin lacks Pseudomonas coverage.

Treatment of FN: High Risk

• Monotherapy

Anti‐Pseudomonal penicillin +/‐ ‐lactamase inhibitor or 

Extended‐spectrum cephalosporin or

Carbapenem

• Combination therapy

Aminoglycoside + anti‐Pseudomonal penicillin +/‐ ‐lactamase inhibitor

Aminoglycoside + extended‐spectrum cephalosporin

Ciprofloxacin + anti‐Pseudomonal penicillin

Referenced with permission from the NCCN Clinical Practice Guidelines in Oncology (NCCN Guidelines®) for Prevention and Treatment of Cancer‐Related Infections V.2.20145© National Comprehensive Cancer Network, Inc 

2015. All rights reserved. Accessed [November 1, 2015].Freifeld AG et al. Clin Infect Dis. 2011; 52:e56‐93.

Question 5: What factors for LK should be considered when deciding if vancomycin should be added to LK’s regimen?

A. All FN patients should receive vancomycin

B. Presence of a central venous catheter

C. Prior ciprofloxacin prophylaxis

D. Reported symptom of cough

FN Treatment: Addition of Vancomycin

• 15 mg/kg IV Q12 hours should be added if Risk factor for Streptococcus viridans

—Prophylaxis (fluoroquinolone or trimethoprim‐sulfamethoxazole)

—Severe mucositis

Colonization with resistant Streptococci or Staphylococci Catheter‐related infection Blood culture positive for gram‐positive bacteria Hypotension/sepsis Skin or Soft‐tissue infection Pneumonia

• Reassess therapy 24‐72 hours after initiation

Referenced with permission from the NCCN Clinical Practice Guidelines in Oncology (NCCN Guidelines®) for Prevention and Treatment of Cancer‐Related Infections V.2.2015 © National Comprehensive Cancer Network, Inc 2015. All rights reserved. Accessed [November 1, 2015].

Freifeld AG et al. Clin Infect Dis. 2011; 52:e56‐93.

©2016 American Society of Health-System Pharmacists, Inc. All rights reserved.

____________________________________________________________________________________________________________ 33

Page 34: 2016 Pharmacotherapy Specialty Examination Review Course ...elearning.ashp.org/Files/Org/c52850f8e2e14ca8b14b... · 2016 Pharmacotherapy Specialty Examination Review Course: Complex

Hospital Course for LK

• Day 1

Respiratory rate improves, blood pressure improves with fluids

LK spikes a fever at 101.9° F

Blood and urine cultures are drawn again

Patient complains of tenderness at port site, which is red on examination

• Broaden Antibiotic Coverage

Imipenem 500 mg IV Q 6 hours

Add vancomycin 15 mg/kg loading dose then 1 g IV Q 12 hours

Check vancomycin trough level prior to 4th dose

Trough goal 15 mcg/ml

Hospital Course for LK• Days 2‐4

LK continues to run low grade fevers (100.3‐101°F)

Vitals remain stable

ANC remains < 0.5 X 109/L

• Day 5

Fever spikes to 103.4° F

Blood pressure drops to 89/54, heart rate 105

Initiate antifungal coverage

Cultures show no growth to date

Next set of blood cultures drawn

ANC rises to 0.5 X 109/L

Labs for LKOn Admission Day 1  Day 2 Day 3 Day 4 Day 5

Na (mEq/L) 138 140 139 138 140 140

K (mEq/L) 4.2 4.1 4.2 4 4.2 4

Cl (mEq/L) 105 102 103 100 100 102

CO2 (mEq/L) 28 28 27 26 28 28

BUN (mg/dL) 25 28 28 30 30 30

SCr (mg/dL) 1.1 1.3 1.3 1.4 1.4 1.3

Glucose (mg/dL) 99 120 121 108 114 115

WBC(X 109/L)

1.7 1.3 1.3 1.2 1.3 1.5

HgB (g/dL) 11.2 11 10.7 10.6 10.3 10

Hct (%) 30 29 28 28 27 27

Plts(X 109/L)

96 90 89 90 71 89

ANC (X 109/L) 0.6 0.4 0.4 0.3 0.4 0.5

Question 6: Which of the following is the best choice for empiric antifungal coverage for LK? 

A. Micafungin 50 mg IV once daily

B. Micafungin 100 mg IV once daily

C. Fluconazole 200 mg IV once daily

D. Voriconazole 6 mg/kg IV every 12 hours

FN Treatment: Addition of Antifungal Coverage

• Most invasive fungal infections are due to Candida or Aspergillosis

• Fungal infections are uncommon early in FN such that treatment is usually delayed

• Candida Therapy

NCCN® – echinocandins

IDSA – insufficient evidence for one agent (FN guidelines); amphotericin B is the “old standard”

• Aspergillosis Therapy

NCCN® ‐ voriconazole

IDSA – voriconazole

• IDSA Guidelines available for Aspergillosis (2008) and Candidiasis(2009)

Referenced with permission from the NCCN Clinical Practice Guidelines in Oncology (NCCN Guidelines®) for Prevention and Treatment of Cancer‐Related Infections V.2.2015 © National Comprehensive Cancer Network, Inc 2015. All rights reserved. Accessed [November 1, 2015].

Freifeld AG et al. Clin Infect Dis. 2011; 52:e56‐93.Pappas PG et al. Clin Infect Dis. 2009; 48:503‐35.   Walsh TJ. Clin Infect Dis. 2008; 46:327‐60.

Approach to a Port‐Related Bloodstream Infection

Mermel LA et al. Clinical practice guidelines for the diagnosis and management of intravascular catheter‐related infection: 2009 update by the Infectious Disease Society of America. Clin Infect Dis. 2009; 49:1‐45. 

Clinical infectious diseases: an official publication of the Infectious Diseases Society of America by INFECTIOUS DISEASES SOCIETY OF AMERICA . Reproduced with permission of Oxford University Press.

Enlarged slide at back of handout.

©2016 American Society of Health-System Pharmacists, Inc. All rights reserved.

____________________________________________________________________________________________________________ 34

Page 35: 2016 Pharmacotherapy Specialty Examination Review Course ...elearning.ashp.org/Files/Org/c52850f8e2e14ca8b14b... · 2016 Pharmacotherapy Specialty Examination Review Course: Complex

Approach to a Port‐Related Bloodstream Infection

Mermel LA et al. Clinical practice guidelines for the diagnosis and management of intravascular catheter‐related infection: 2009 update by the Infectious Disease Society of America. Clin Infect Dis. 2009; 49:1‐45. 

Clinical infectious diseases: an official publication of the Infectious Diseases Society of America by INFECTIOUS DISEASES SOCIETY OF AMERICA . Reproduced with permission of Oxford University Press.

Management of Candidemia

• Remove the long‐term catheter (port)

• An echinocandin should be used to treat critically‐illpatients until fungal isolate is identified

• Fluconazole may be used empirically for thefollowing patients

No azole exposure in the previous 3 months

Setting with low risk of C. krusei or C. glabrata

Mermel LA et al.  Clin  Infect Dis.  2009; 49:1‐45.

Treatment Options for Candida

Drug DoseDosage 

AdjustmentsRecommendation

Fluconazole400 mg daily

Renal impairment

If susceptible

Caspofungin 70 mg loading dose then 50 mg 

daily

Hepatic Impairment

Preferred

Anidulafungin 200 mg loading dose then 100 mg 

dailyNone Preferred

Micafungin 100 mg daily None Preferred

LiposomalAmphotericin B

3‐5 mg/kg daily None Alternative

Note: all agents are intravenous except fluconazole which is also available as an oral formulation.

Referenced with permission from the NCCN Clinical Practice Guidelines in Oncology (NCCN Guidelines®) for Prevention and Treatment of Cancer‐Related Infections V.2.2015 © National Comprehensive Cancer Network, Inc 

2015. All rights reserved. Accessed [November 1, 2015].

Culture Results for LK

Blood cultures(unless otherwise specified)

Admit Peripheral No Growth

Admit Central No Growth

Urine C & S 1+ mixed Gram‐positive flora

Day 2 Central No Growth

Day 2 Peripheral No Growth

Day 5 Central 4 + Yeast

Day 5 Peripheral 3 + Yeast

Hospital Course for LK• Days 6 – 8

No further fevers

Vitals stabilized

ANC recovers to > 1 X 109/L

• Day 9

Fungal cultures return positive for C. albicans, sensitive to fluconazole

Discontinue imipenem, vancomycin

Follow Up for LK

• LK was discharged and completed her course offluconazole without complications

• Her port was removed and she refused replacement;the remainder of her chemotherapy was given peripherally (carefully)

• Successfully completes chemotherapy without incidence

Last week of paclitaxel was not given due to peripheral neuropathy

• Initiated on adjuvant anastrozole 1 mg PO daily (planfor 5 years of therapy)

©2016 American Society of Health-System Pharmacists, Inc. All rights reserved.

____________________________________________________________________________________________________________ 35

Page 36: 2016 Pharmacotherapy Specialty Examination Review Course ...elearning.ashp.org/Files/Org/c52850f8e2e14ca8b14b... · 2016 Pharmacotherapy Specialty Examination Review Course: Complex

Endocrine Therapy Medication Adherence

• Data evaluating adherence are variable

Methods

Definitions

Endpoints

Duration of follow up

• Some studies show up to 50% discontinuation rates of endocrine therapies prior to completion of recommended course

• Discontinuation is linked to increased mortalityHershman DL et al. Breast Cancer Res Treat. 2011; 126: 529‐37.

McCowan C et al. Br J Cancer. 2008; 99:1763‐8.Dezentje VO et al. J Clin Oncol. 2010; 28(14):2423‐9.Hershman DL et al. J Clin Oncol. 2010; 28(27):4120‐8.

Barriers to Adherence: Adjuvant Aromatase Inhibitor Therapy

Patient‐specific Factors Provider‐related Factors Treatment‐related Factors

Health beliefs (lack of belief in therapy)

Importance of adherence may not be emphasized

Adverse effects

Depression/Antidepressant use Difficulties of long term adherence may not be discussed

Underestimation of side effects by physician

Mastectomy Lack of support for dealing with side effects

Unexpected effects associated with non‐adherence

Poor awareness of therapeutic benefit

Dissatisfaction with provider

Remembering to take medication(Older and younger age groups associated with non‐adherence)

Inconvenience

Difficulty swallowing pills

Verma S et al. Curr Oncol. 2011 May; 18 Suppl 1:S3‐9.

Fast forward 4 years…

• LK has remained adherent to her anastrozole therapy

• In January 2017, her primary care physician startedher on paroxetine 20 mg orally daily as LK became more socially isolated, had difficulty sleeping and worried constantly about her cancer coming back

• In June 2020, she presents to the Emergency Department with sudden, severe, new onset chestand lower back pain.  She is short of breath (RR 25)and tachycardic (HR 109).

Labs• CK Index 1• CK‐MB 1 ng/ml• Total CK 41 ng/ml• Troponin 0.2 ng/ml

• WBC 5.3 thou/microL• Hgb 13.2 g/dL• Hct 32%• Plts 144 thou/microL• ANC 2.7 thou/microL

• Na 135 mEq/L• K 4.0 mEq/L• Cl 100 mEq/L• CO2 30 mEq/L• BUN 17 mg/dL• SCr 0.87 mg/dL• Glucose 99 mg/dL

• AST 25 units/L• ALT 18 units/L• Alk Phos 105 units/L• T bili 1.0 mg/dL• TP 7.1 g/dL• Albumin 3.9 g/dL

• Spiral CT reveals bilateral pulmonary embolism

Small pulmonary embolus in the distal right lower lobe arterial tree

Large pulmonary embolus in the left upper lobe

• MRI Thoracic and Lumbar Spine

Compression fractures at T11, L1 and L2

Multiple small lesions throughout T7 to L3

• LDH 456 units/L

• Calcium 10.5 mg/dL

Tests Question 7: Which of the following are risk factors for venous thromboembolism (VTE) in LK?

A. Metastatic Breast Cancer

B. Nausea and Vomiting

C. Presence of Central Venous Catheter 

D. Depression

©2016 American Society of Health-System Pharmacists, Inc. All rights reserved.

____________________________________________________________________________________________________________ 36

Page 37: 2016 Pharmacotherapy Specialty Examination Review Course ...elearning.ashp.org/Files/Org/c52850f8e2e14ca8b14b... · 2016 Pharmacotherapy Specialty Examination Review Course: Complex

VTE in Malignancy

• Endothelial injury

Central catheter

Chemotherapy

Surgery

• Circulatory Stasis

Prolonged bed rest

Vascular invasion by tumor cells

• Hypercoagulable state

Procoagulants from tumor

Release of cytokines

Direct cell‐to‐cell interaction between endothelial cells, leukocytes and platelets

Question 8:  Which of the following therapies should be initiated in LK for her pulmonary embolism?

A. Warfarin 5 mg orally daily

B. Apixaban 10 mg orally twice daily

C. Dalteparin 12,500 units subcutaneous every 24 hr

D. Enoxaparin 80 mg subcutaneous every 12 hr

Low Molecular Weight Heparin (LMWH) vs. a Coumarin (CLOT Study)

• All patients initiated on dalteparin subcutaneous 200 units/kg/day

• N = 336 continued on dalteparin subcutaneous 200 units/kg/day X 1 month then dalteparin subcutaneous 150 units/kg/day X 5 months

• N = 336 received dalteparin subcutaneous X 5‐7 days with a coumarin derivative orally X 6 months with target     INR = 2.5

• Primary outcome: Recurrent DVT and/or PE

• Secondary outcomes: Clinically overt bleeding, death

Lee AYY et al. N Engl J Med. 2003; 349:146‐53.

CLOT Results

• Primary outcome

Recurrence rates at 6 months: 8% in dalteparin group vs. 16% in the warfarin group

• Secondary outcomes

No significant difference in the rate of major bleeding (6% in the dalteparin group vs. 4% in the oral anticoagulant group)

Mortality rate at 6 months was 39% vs. 41% in the dalteparin and oral anticoagulant arms

Lee AYY et al. N Engl J Med. 2003; 349:146-53.

CATCH

• N= 449 Tinzaparin 175 IU/kg subcutaneous once daily X 6 months

• N= 451 Tinzaparin 175 IU/kg subcutaneous once daily X 5‐10 days then warfarin orally once daily (goal INR 2‐3) X 6 months

• Primary outcome: Recurrent thrombosis

6‐month cumulative incidence, 7.2% for tinzaparin vs 10.5% for warfarin; HR 0.65 [95%CI, 0.41‐1.03]; P = .07

• Secondary safety outcomes: major bleeding, clinically relevant nonmajor bleeding, and overall mortality

Significant reduction in clinically relevant nonmajor bleeding with tinzaparin (49 of 449 patients for tinzaparinvs 69 of 451 patients for warfarin; HR, 0.58 [95%CI, 0.40‐0.84]; P = .004) Lee AYY, et al. JAMA. 2015;314(7):677‐686.

Thrombosis in Malignancy:Current Recommendations• Initial Phase (5‐7 days)

Dalteparin 200 units/kg subcutaneous Q 24 hours OR

Enoxaparin 1 mg/kg subcutaneous Q 12 hours

• Subacute Phase (3‐6 months)

Dalteparin 200 units/kg subcutaneous Q 24 hours X 1 month then 150 units/kg Q 24 hours OR

Enoxaparin 1 mg/kg subcutaneous Q 12 hours or 1.5 mg/kg Q 24 hours

• Chronic Phase (6 months to indefinite)

Continue anticoagulation long term or until malignancy resolves

— LMWH

— Warfarin with INR 2‐3

Kearon C et. al. Chest 2012; 141(2 suppl):419S‐94S. Referenced with permission from the NCCN Clinical Practice Guidelines in Oncology (NCCN Guidelines®) for Venous Thromboembolic Disease 

V.1.2015 © National Comprehensive Cancer Network, Inc 2015. All rights reserved. Accessed [November, 2015].

©2016 American Society of Health-System Pharmacists, Inc. All rights reserved.

____________________________________________________________________________________________________________ 37

Page 38: 2016 Pharmacotherapy Specialty Examination Review Course ...elearning.ashp.org/Files/Org/c52850f8e2e14ca8b14b... · 2016 Pharmacotherapy Specialty Examination Review Course: Complex

LK’s ED Course

• LK is initiated on enoxaparin 80 mg subcutaneous Q12 hours

She demonstrates the ability to give herself injections

LK is counseled on signs and symptoms of over and under anticoagulation

A referral for follow up at the cancer center’s pharmacist‐run anticoagulation clinic is made

—Monitor for heparin‐induced thrombocytopenia (HIT) (platelet checks)

—Assess for symptoms of improvement

LK’s ED Course

• Vitals at hour 3 in the ED

RR 20, BP 130/82, HR 100, Temp 98.1°F

• LK completes a NS 500 mL fluid bolus

• Hydromorphone 2 mg IV X 2 is administered, patientreports improvement of back pain

• She reports resolution of shortness of breath and chestpain

• For pain control she is prescribed hydromorphone 1 mgorally every 4 hours as needed

• Follow up with her oncologist for breast cancer recurrence

Interval History• LK comes to clinic 1 week after her ED visit to discusstherapy options for her recurrent breast cancer

• She reports adherence to enoxaparin, but “hates the shots”

Her oncologist says that they can discuss changing to warfarin after 6 months of injections

• Platelet count, renal function are stable on labs

• She continues to struggle with pack pain

The pain is worse at night as she cannot sleep comfortably

She has been taking two tablets of hydrocodone at 8am, noon, 4pm but three hydrocodone tablets at bedtime and 2‐3 times per night when she wakes up

Question 9: LK’s oncologist asks you for a conversion to a long‐acting opioid as LK has escalated her hydrocodone use and is still reporting poor pain control with inability to sleep.  Which of the following is the best regimen for LK?

A. Fentanyl TDS 100 mcg/hr every 72 hours

B. Morphine SR 15 mg poQ 8 hours

C. Morphine SR 15 mg poQ 12 hours

D. Oxycodone SR 30 mg po Q 12 hours

Pain

• “an unpleasant sensory and emotional experience associated with actual or potential tissue damage, or described in such terms”

International Association for the Study of Pain 1994

• Whatever the patient says it is

Prevalence of Types of Cancer Pain (Pathophysiology)

Pain Type Percentage

Somatic 32‐35%

Visceral 15‐17%

Neuropathic 8‐9%

Somatic & Visceral 11‐13%

Somatic & Neuropathic 21‐23%

Visceral & Neuropathic 2‐4%

Breakthrough 65%

Chang VT et al. J Palliat Med. 2006; 9:1414‐34.

©2016 American Society of Health-System Pharmacists, Inc. All rights reserved.

____________________________________________________________________________________________________________ 38

Page 39: 2016 Pharmacotherapy Specialty Examination Review Course ...elearning.ashp.org/Files/Org/c52850f8e2e14ca8b14b... · 2016 Pharmacotherapy Specialty Examination Review Course: Complex

World Health Organization (WHO) Principles of Pharmacologic Management

• By the mouth

• By the clock

• By the ladder

• For the individual

• With attention to detail

World Health Organization 1996; 2nd Edition.

Adapted from WHO Pain Relief Ladder.Palliative Pharmacy Care edited by Jennifer Strickland, published in 2009 by ASHP.

“Strong” opioids+/‐ APAP/NSAIDs+/‐ adjuvants

“Weak” opioids+/‐ APAP/NSAIDs+/‐ adjuvants

Acetaminophen+/‐ NSAIDs+/‐ adjuvants

APAP = acetaminophen

“Weak” opioids: e.g., codeine, hydrocodone, oxycodone/APAP“Strong” opioids: e.g., morphine, hydromorphone, oxycodone

WHO Pain Relief Ladder

Therapy Options• Non‐Opioid

Acetaminophen (APAP)

NSAIDs

COX‐2 Selective Agents

• Opioids

See upcoming slides

• Adjuvants

Note: Use is off‐label Antidepressants Anticonvulsants Consider risk of suicide

Local Anesthetics Bisphosphonates Calcitonin Corticosteroids Hydroxyzine

Pereira A et al. Pain 2013; 154:345‐9.

Opioids: What to UseMild‐Moderate Pain

• Hydrocodone/APAP*

• Oxycodone/APAP*

• Codeine

• *Combo products have variable amounts of opioid

Severe Pain

• Morphine

• Oxycodone

• Hydromorphone

• Methadone

• Fentanyl

• Oxymorphone

Opioids: Essential Considerations

• Medication allergies

• Previous opioid exposure and preference

• Severity and nature of disease

• Age of patient

• Extent of cancer, particularly hepatic and renalinvolvement altering opioid pharmacokinetics

• Comorbid disease states

• Dosage form and route of administration

Opioid Dose Titration• Patients currently on opioids

No consensus or guidelines available

• Severe pain (7‐10)** Consider increasing dose by 50‐100%

• Moderate pain (4‐6)** Consider increasing dose by 25‐50%

• Minimal pain (1‐3)** No increase in dose if controlled with current breakthrough pain (BTP) medication

Consider increasing dose by 25%

• End‐of‐dose failure Shorten dosing interval

**On a 10‐point scale with 10 for the worst pain imaginable for the three severity ratings on this slide

©2016 American Society of Health-System Pharmacists, Inc. All rights reserved.

____________________________________________________________________________________________________________ 39

Page 40: 2016 Pharmacotherapy Specialty Examination Review Course ...elearning.ashp.org/Files/Org/c52850f8e2e14ca8b14b... · 2016 Pharmacotherapy Specialty Examination Review Course: Complex

Opioids: Equianalgesic Dosing

• Incomplete cross tolerance between opioids

• Wide interindividual variability

• Several different equianalgesia charts available Derived from single‐dose studies

Do not reflect long‐term opioid exposure or changes made due to ineffective analgesia or toxicity

• If pain poorly controlled, use aggressive conversion 75‐100% of dose

• If pain well controlled, use conservative conversion 50‐75% of dose

Opioids: Equianalgesic DosingOPIOID ORAL DOSE PARENTERAL DOSE

Codeine 200 mg N/A

Hydrocodone 30 ‐ 45 mg N/A

Oxycodone 15‐20 mg N/A

Morphine 30 mg 10 mg

Hydromorphone 7.5 mg 1.5 mg

Methadone* * *

Fentanyl** N/A 100 mcg

Oxymorphone 10 mg 1 mg

*Ratio may range from 1:1 at low doses of oral morphine up to 20:1 for patients receiving high doses of oral morphine (~300mg/day)**Fentanyl transdermal systems 25mcg/hr  60mg PO morphine 

Referenced with permission from the NCCN Clinical Practice Guidelines in Oncology (NCCN Guidelines®) for Adult Cancer Pain V.2.2015 © National Comprehensive Cancer Network, Inc 2015. All rights reserved. Accessed 

[November 1, 2015].

Stepwise Approach to Conversion

1. Comprehensive pain assessment

Current level of pain control

Functional status

2. Determine total daily opioid use

3. Use conversion table to convert to new opioid               

4. Individualize dose

More aggressive conversion for uncontrolled pain

Less aggressive conversion for well controlled pain

5. Follow‐up and continual reassessment 

Question 9 Revisited: LK’s oncologist asks you for a conversion to a long‐acting opioid as LK has escalated her hydrocodone use and is still reporting poor pain control with inability to sleep.  Which of the following is the best regimen for LK?

A. Fentanyl TDS 100 mcg/hr every 72 hours

B. Morphine SR 15 mg po Q 8 hours

C. Morphine SR 15 mg po Q 12 hours

D. Oxycodone SR 30 mg po Q 12 hours

Question 10: LK’s oncologist follows your advice and initiates the long acting pain regimen that was recommended. Which of the following is the best breakthrough pain regimen for her?

A. Morphine IR 15 mg poevery 1‐2 hours

B. Morphine IR 15 mg poevery 4‐6 hours

C. Morphine IR 30 mg poevery 6 hours

D. Oxycodone 2.5 mg poevery 6 hours

Breakthrough Pain (BTP)

• A fluctuation in pain intensity that interrupts a tolerable background pain.

• 3 types Idiopathic

Incident (predictable and unpredictable)

End‐of‐dose failure

• Characterized by Moderate to severe intensity

Rapid onset (<3 minutes)

Relatively short duration (< 1 hour)

Frequency averaging 1‐6 episodes per day

©2016 American Society of Health-System Pharmacists, Inc. All rights reserved.

____________________________________________________________________________________________________________ 40

Page 41: 2016 Pharmacotherapy Specialty Examination Review Course ...elearning.ashp.org/Files/Org/c52850f8e2e14ca8b14b... · 2016 Pharmacotherapy Specialty Examination Review Course: Complex

Breakthrough Pain• The Ideal BTP Medication

Rapid onset (immediate release)

Easy to titrate

Short duration

Cost effective

Minimal side effects

• PO Dose: 10‐20% of daily dose q3hr PRN, OR

• PRN dose given as immediate release product during a given interval equals the total scheduled dose of a sustained release product given in the same interval e.g., morphine SR 90mg PO Q12h = morphine IR 30mg PO q4h 

Cancer‐Related Neuropathic Pain

• NCCN® Guidelines

Antidepressants and anticonvulsants are 1st line adjuvant medications

Derived from data in non‐cancer patients

• ASCO Guidelines for CIPN

No agents recommended for prevention

Moderate recommendation for treatment with duloxetine

• N=231

• 5 week treatment course: duloxetine 30 mg X 1 week then 60 mg X 4 weeks vs. placebo

• Primary outcome: average pain intensity (0‐10 scale)

Duloxetine arm: mean decrease in average pain of 1.06 (95% CI, 0.72‐1.40) vs. placebo arm: 0.34 (95% CI, 0.01‐0.66) (P = .003; effect size, 0.513)

CIPN: Chemotherapy-induced peripheral neuropathyHershman DL, et al. J Clin Oncol 2014; 32:1941-1967.

Smith EM et al. JAMA. 2013 Apr 3;309(13):1359-67.National Comprehensive Cancer Network. NCCN® clinical practice guidelines in oncology: adult cancer pain. Version 2.2015.

Referenced with permission from the NCCN Clinical Practice Guidelines in Oncology (NCCN Guidelines®) for Adult Cancer Pain V.2.2054 © National Comprehensive Cancer Network, Inc 2015. All rights reserved. Accessed November 1, 2015].

Breast Cancer Epidemiology

• 231,840 new cases of invasive breast cancer expected in 2015

• 40,290 women expected to die from breast cancer this year

• Incidence rates decreased 7% from 2002‐2003

Women’s Health Initiative published in 2002

Decline in use of hormone replacement therapy

Incidence rates stable recently

• 2nd leading cause of cancer death in women

• 2.8 million survivors in the United States

American Cancer Society. Available at: http://www.cancer.org/cancer/breastcancer/detailedguide/breast-cancer-key-statistics. February 8,2016.

Individualized Drug Therapy in Breast Cancer• Menopausal Status

Potential for use of tamoxifen vs. aromatase inhibitors

Need for ovarian suppression or ablation

Fertility preservation

• Hormone Receptor Status (ER/PR)

Use of hormonal therapies

• HER2 Status

Use of trastuzumab, lapatinib, pertuzumab or ado‐trastuzumab emtansine

• Patient factors such as performance status, organ function, etc.

Hormonal Therapies

• Nonsteroidal Aromatase Inhibitors

Anastrozole

Letrozole

• Steroidal aromatase inactivator  Exemestane

• Selective Estrogen Receptor Modulators

Tamoxifen

Toremifene

• Estrogen Receptor Antagonist  Fulvestrant

• Others include megestrol acetate, fluoxymesterone, ethinyl estradiol

Palbociclib (Ibrance®)• Indication

In combination with letrozole in postmenopausal patients with ER+Her2‐metastatic breast cancer (1st line therapy)

In combination with fulvestrant in patients with ER+Her2‐ advanced/metastatic breast cancer after progression on endocrine therapy

• MOA

Inhibits cyclin‐dependent kinase (CDK) 4 & 6

Deceases cell proliferation by blocking cell progression from G1 to S phase

• Dose: 125 mg po daily Days 1‐21 Q 28 days

• With letrozole: Median PFS 20.2 months (95% CI 13.8‐27.5) vs. 10.2 months (95% CI 5.7‐12.6) with letrozole alone, HR 0.488 (95% CI 0.319‐0.784)

• With fulvestrant: Median PFS 9.2 months (95% CI, 7.5 to not estimable) vs. 3.8 months (95% CI, 3.5 to 5.5) with fulvestrant alone, HR 0.42 (95% CI, 0.32 to 0.56; P<0.001)

Finn RS et al. Lancet Oncol. 2015; 16: 25-35.Turner NC et al. N Engl J Med. 2015:373: 209-19.

©2016 American Society of Health-System Pharmacists, Inc. All rights reserved.

____________________________________________________________________________________________________________ 41

Page 42: 2016 Pharmacotherapy Specialty Examination Review Course ...elearning.ashp.org/Files/Org/c52850f8e2e14ca8b14b... · 2016 Pharmacotherapy Specialty Examination Review Course: Complex

Active Antineoplastic Agents in Breast Cancer

Class Agents

Anthracyclines

Doxorubicin

Epirubicin

Liposomal doxorubicin

Taxanes

Paclitaxel

Docetaxel

Albumin‐bound paclitaxel

AntimetabolitesGemcitabine

Capecitabine

Other Microtubule InhibitorsVinorelbine

Eribulin

Other Active AgentsCyclophosphamide, mitoxantrone, cisplatin, carboplatin, etoposide, vinblastine, fluorouracil, ixabepilone

HER2 Targeted Therapy

• Trastuzumab Adjuvant Setting

—as part of a treatment regimen consisting of doxorubicin, cyclophosphamide, and a taxane

—with docetaxel and carboplatin

—single agent following multi‐modality anthracycline‐based therapy

Metastatic Setting—In combination with paclitaxel for 1st line treatment 

—As a single agent for patients who have received one or more chemotherapy regimens for metastatic disease

HER2 Targeted Therapy• Lapatinib

With capecitabine, for the treatment of patients who have received prior therapy including an anthracycline, a taxane, and trastuzumab

With letrozole

• Pertuzumab

In combination with trastuzumab and docetaxel for patients with metastatic breast cancer who are treatment‐naïve

Neoadjuvant therapy (with trastuzumab and docetaxel) for early breast cancer

• Ado‐trastuzumab emtansine

As a single agent, for the treatment of patients who previously received trastuzumab and a taxane (separately or in combination)

CLEOPATRA

• Trastuzumab and docetaxel +/‐ pertuzumab in HER2+metastatic breast cancer with no prior treatment

• Median overall survival was 56.5 months (95% CI, 49.3 to not reached) in the pertuzumab arm, compared with 40.8 months (95% CI, 35.8 to 48.3) in the placebo arm (HR favoring the pertuzumab group,0.68; 95% CI, 0.56 to 0.84; P<0.001)

• 15.7 months difference

Swain SM, et al. N Engl J Med. 2015; 372:724-34.

Question 11: Which of these therapies is a possible treatment option for LK? 

A. Palbociclib and fulvestrant

B. Ado‐trastuzumabemtansine

C. Pertuzumab, trastuzumab and paclitaxel

D. Palbociclib and docetaxel

Follow‐up Therapy for Endocrine Treatment of Recurrent or Stage IV Disease

• Continue endocrine therapy until progression or unacceptable toxicity

• If no clinical benefit after 3 sequential endocrine regimens OR

• Symptomatic visceral disease 

Proceed with chemotherapy

Referenced with permission from the NCCN Clinical Practice Guidelines in Oncology (NCCN Guidelines®) for Breast Cancer V.1.2016 © National Comprehensive Cancer Network, Inc 2016. All rights reserved. Accessed [March 6, 2016].

©2016 American Society of Health-System Pharmacists, Inc. All rights reserved.

____________________________________________________________________________________________________________ 42

Page 43: 2016 Pharmacotherapy Specialty Examination Review Course ...elearning.ashp.org/Files/Org/c52850f8e2e14ca8b14b... · 2016 Pharmacotherapy Specialty Examination Review Course: Complex

Examples of Safety Issues in Our Patient Case• Medication Adherence

Hormonal therapies (other oral therapies)

• Safe use of anticoagulant therapy

Warfarin drug‐drug interactions

—Consider influence of active cancer and chemotherapy on bleeding risk

• Extravasation

LK’s need for a central venous catheter

• Cardiotoxicity

Anthracycline (others include HER2 agents)

Ambulatory Health Care National Patient Safety Goal (NPSG.03.05.01) 

• Reduce the likelihood of patient harm associated with the use of anticoagulant therapy

• Applies to organizations that provide anticoagulant therapy and/or long‐term anticoagulation prophylaxis where the clinical expectation is that the patient’s laboratory values for coagulation will remain outside normal values

• Elements of performance

Use approved protocols for the initiation and maintenance of anticoagulant therapy

Before starting a patient on warfarin, assess the patient’s baseline coagulation status; for all patients receiving warfarin therapy, use a current INR to adjust this therapy

Provide education regarding anticoagulant therapy to prescribers, staff, patients, and families

Evaluate anticoagulation safety practices, take action to improve practices, and measure the effectiveness of those actions in a time frame determined by the organization

Available at: http://www.jointcommission.org/assets/1/6/2016_NPSG_AHC.pdf.  Accessed February 28, 2016.  

Warfarin Drug Interactions• Too many for this slide!

Substrate of CYP1A2, 2C19, 2C9 (major), and 3A4

Weak inhibitor of CYP2C19 and 2C9

Common agents for cancer patients (e.g., antibiotics, chemotherapy)

• Increased bleeding risk

Thrombocytopenia

Disease related

Chemotherapy

Bleeding tumors

• Intensive monitoring required for cancer patients

Changes in appetite

Fatigue

Nausea/vomiting

Example: increased fall risk due to chemotherapy‐induced peripheral neuropathy

Extravasation• Symptoms can be delayed or immediate

Pain, burning

Swelling

Erythema

Loss of blood return

• Determine if agent binds to DNA for direct vs. indirect effect

• Results in partial or full thickness skin loss

• Minimal evidence base for treatment

Schulmeister L. J Clin Oncol Nurs. 2007; 11(3):387‐95.  Wickham R et al. Oncology Nursing Forum. 2006; 33:1143‐50.

Schrijvers DL. Annals of Oncology. 2003; 14 (supplement 3): iii26‐30.Photos from: N Engl J Med, Vano‐Galvan S, Jaen P, Extravasation of Epirubicin, 360, 

2117. Copyright © 2009 Massachusetts Medical Society. Reprinted with permission from Massachusetts Medical Society.    

Treatment of ExtravasationAgent Antidote Comments

Anthracyclines Dexrazoxane1000 mg/m2 IV within 5 hours on Day 1, repeat Day 2 then 500 mg/m2 IV on Day 3

DMSO Apply locally ASAP and repeat Q 8 hours X 7 days

Cold packs Vasoconstriction decreases local drug uptake

Nitrogen Mustards Sodium thiosulfate2 mL of a solution of 4 mL sodium thiosulfate and 6 mL sterile water

Vinca Alkaloids Hyaluronidase 50‐1500 units subcutaneous

Warm Packs Vasodilation to enhance drug uptake

Taxanes Normal saline Exerts dilutional effect

DMSO

Cold packs ?

Surgical intervention may also be warranted

Wickham R et al. Oncology Nursing Forum. 2006; 33:1143‐50.Schrijvers DL.  Annals of Oncology. 2003; 14 (supplement 3): iii26‐30.

Cardiotoxicity

• Anthracycline Induced ‐ Cardiomyopathy

Delayed

Cumulative

Dose‐related (Lifetime maximum dose)

Risk factors include heart disease, HTN, mediastinal radiation, prior anthracycline

• Trastuzumab Induced

Additive with anthracyclines

• Recommendations for monitoring for our patient’s initial chemotherapy regimen

Cardiac monitoring at baseline, 3, 6 and 9 months

©2016 American Society of Health-System Pharmacists, Inc. All rights reserved.

____________________________________________________________________________________________________________ 43

Page 44: 2016 Pharmacotherapy Specialty Examination Review Course ...elearning.ashp.org/Files/Org/c52850f8e2e14ca8b14b... · 2016 Pharmacotherapy Specialty Examination Review Course: Complex

Pharmacist’s Role in Complex Oncology Patients

• Drug therapy recommendations

Chemotherapy

Supportive care

• Anticoagulation management

• Pain Management

• Medication safety, especially chemotherapy

• Medication access

• Monitoring 

• Patient and family education

Cancer Patient Resources

• NCCN® Guidelines for Patients                                        (http://www.nccn.org/patients/default.asp) Referenced with permission from the NCCN Clinical Practice Guidelines in Oncology (NCCN Guidelines®) for Patients © National Comprehensive Cancer Network, Inc 2014. All rights reserved. Accessed [November 19, 2014].

• ASCO Website for Patients (http://www.cancer.net)

• American Cancer Society (http://www.cancer.org/)

• LIVESTRONG (http://www.livestrong.org/)

• Clinical Trials (www.clinicaltrials.gov)

©2016 American Society of Health-System Pharmacists, Inc. All rights reserved.

____________________________________________________________________________________________________________ 44

Page 45: 2016 Pharmacotherapy Specialty Examination Review Course ...elearning.ashp.org/Files/Org/c52850f8e2e14ca8b14b... · 2016 Pharmacotherapy Specialty Examination Review Course: Complex

Approach to a Port‐Related Bloodstream Infection

Mermel LA et al. Clinical practice guidelines for the diagnosis and management of intravascular catheter‐related infection: 2009 update by the Infectious Disease Society of America. Clin Infect Dis. 2009; 49:1‐45. 

Clinical infectious diseases: an official publication of the Infectious Diseases Society of America by INFECTIOUS DISEASES SOCIETY OF AMERICA . Reproduced with permission of Oxford University Press.

©2016 American Society of Health-System Pharmacists, Inc. All rights reserved.

____________________________________________________________________________________________________________ 45